Sunteți pe pagina 1din 164

.

Universidade Federal de Alagoas Instituto de Matem atica


Curso de Licenciatura em Matem atica - UAB Disciplinas: Algebra Elementar e Elementos de Matem atica 1 Autores: Amauri da Silva Barros Ediel Azevedo Guerra e Paulo Roberto Lemos

Conte udo
1 Equa co es do 1 grau a uma inc ognita 1.1 1.2 1.3 Aprendendo uma linguagem simb olica . . . . . . . . . . . . . . . . . . . . . . O m etodo da Inspe ca o Direta (ou das Tentativas) . . . . . . . . . . . . . . . O M etodo Operat orio . . . . . . . . . . . . . . . . . . . . . . . . . . . . . . . 1.3.1 1.4 A exist encia de inversos . . . . . . . . . . . . . . . . . . . . . . . . . 6 6 7 10 10 11 12 15 15 21 23 24 26 28 30 31 31 32 35

As propriedades associativas da adi ca o e da multiplica ca o. . . . . . . . . . . 1.4.1 Os princ pios aditivo e multiplicativo . . . . . . . . . . . . . . . . . .

2 Equa co es do 1 Grau a Uma Inc ognita 2.1 2.2 Resolu ca o de alguns problemas . . . . . . . . . . . . . . . . . . . . . . . . . O que e resolver uma equa ca o? . . . . . . . . . . . . . . . . . . . . . . . . .

3 Produtos Not aveis 3.1 3.2 3.3 3.4 O quadrado de uma soma . . . . . . . . . . . . . . . . . . . . . . . . . . . . O quadrado de uma diferen ca . . . . . . . . . . . . . . . . . . . . . . . . . . A diferen ca de quadrados . . . . . . . . . . . . . . . . . . . . . . . . . . . . . O cubo de uma soma . . . . . . . . . . . . . . . . . . . . . . . . . . . . . . .

4 Fatora c ao 4.1 4.2 4.3 Fatora ca o de um n umero . . . . . . . . . . . . . . . . . . . . . . . . . . . . . Fatora ca o de um n umero em fatores primos . . . . . . . . . . . . . . . . . . Fatora ca o de Mon omios . . . . . . . . . . . . . . . . . . . . . . . . . . . . .

4.4 4.5

Fatora ca o de Bin omios . . . . . . . . . . . . . . . . . . . . . . . . . . . . . . Fatora ca o de Trin omios . . . . . . . . . . . . . . . . . . . . . . . . . . . . . .

37 39 42 43 44 44

5 Equa co es Quadr aticas 5.1 5.2 Equa co es quadr aticas incompletas . . . . . . . . . . . . . . . . . . . . . . . . Equa co es quadr aticas completas . . . . . . . . . . . . . . . . . . . . . . . . . 5.2.1 5.2.2 Casos em que o trin omio ax2 + bx + c e um quadrado perfeito. . . . . Casos em que o trin omio ax2 + bx + c n ao e um quadrado perfeito (completamento de quadrado) . . . . . . . . . . . . . . . . . . . . . . 5.3 Fatora ca o de trin omios . . . . . . . . . . . . . . . . . . . . . . . . . . . . . .

46 50 51 51 54 55 57 60 60 61 61 62 62 62 63 64 66 67 70

6 Fra co es Alg ebricas 6.1 6.2 6.3 6.4 Fra co es num ericas . . . . . . . . . . . . . . . . . . . . . . . . . . . . . . . . . Fra co es alg ebricas . . . . . . . . . . . . . . . . . . . . . . . . . . . . . . . . . Simplica ca o de fra co es alg ebricas . . . . . . . . . . . . . . . . . . . . . . . . Adi ca o de fra co es alg ebricas . . . . . . . . . . . . . . . . . . . . . . . . . . .

7 Polin omios 7.1 7.2 Conceito de polin omio . . . . . . . . . . . . . . . . . . . . . . . . . . . . . . Opera co es com polin omios . . . . . . . . . . . . . . . . . . . . . . . . . . . . 7.2.1 7.2.2 7.2.3 7.2.4 7.2.5 7.2.6 7.3 7.4 7.5 Adi ca o de polin omios . . . . . . . . . . . . . . . . . . . . . . . . . . . O produto de um n umero e de um polin omio . . . . . . . . . . . . . . A diferen ca de polin omios . . . . . . . . . . . . . . . . . . . . . . . . O produto de dois polin omios . . . . . . . . . . . . . . . . . . . . . . O polin omio identicamente nulo . . . . . . . . . . . . . . . . . . . . . Quociente de polin omios . . . . . . . . . . . . . . . . . . . . . . . . .

Raiz ou zero de um polin omio . . . . . . . . . . . . . . . . . . . . . . . . . . Divisibilidade de polin omios . . . . . . . . . . . . . . . . . . . . . . . . . . . O resto da divis ao de um polin omio por X r . . . . . . . . . . . . . . . . .

7.6 7.7

Determina ca o das ra zes racionais de um polin omio . . . . . . . . . . . . . . Rela co es entre coecientes e ra zes . . . . . . . . . . . . . . . . . . . . . . . .

71 76 80 80 81 82 83 84 85 86 87 89 90 91 91 92 94 95 96 96 97 99

8 Conjuntos Num ericos 8.1 8.2 Conjuntos: algumas no co es preliminares . . . . . . . . . . . . . . . . . . . . O conjunto dos n umeros naturais . . . . . . . . . . . . . . . . . . . . . . . . 8.2.1 8.3 Propriedades operat orias dos n umeros naturais . . . . . . . . . . . . .

O conjunto dos n umeros inteiros . . . . . . . . . . . . . . . . . . . . . . . . . 8.3.1 8.3.2 O valor absoluto de um n umero . . . . . . . . . . . . . . . . . . . . . Propriedades operat orias dos n umeros inteiros . . . . . . . . . . . . .

8.4

O conjunto dos n umeros racionais . . . . . . . . . . . . . . . . . . . . . . . . 8.4.1 8.4.2 Opera co es com os n umeros racionais . . . . . . . . . . . . . . . . . . Decimais peri odicos . . . . . . . . . . . . . . . . . . . . . . . . . . . .

8.5 8.6

O conjunto dos n umeros irracionais . . . . . . . . . . . . . . . . . . . . . . . O conjunto dos n umeros reais . . . . . . . . . . . . . . . . . . . . . . . . . . 8.6.1 8.6.2 8.6.3 A reta real . . . . . . . . . . . . . . . . . . . . . . . . . . . . . . . . . Intervalos . . . . . . . . . . . . . . . . . . . . . . . . . . . . . . . . . Valor absoluto ou m odulo de um n umero real . . . . . . . . . . . . .

8.7

Produto cartesiano de conjuntos . . . . . . . . . . . . . . . . . . . . . . . . .

9 Fun c ao Am 9.1 9.2 9.3 9.4 9.5 9.6 9.7 Conceitos de Fun ca o e de Fun ca o Am . . . . . . . . . . . . . . . . . . . . . Deni ca o de Fun ca o e Exemplos . . . . . . . . . . . . . . . . . . . . . . . . . Gr aco de uma Fun ca o . . . . . . . . . . . . . . . . . . . . . . . . . . . . .

Fun ca o Am . . . . . . . . . . . . . . . . . . . . . . . . . . . . . . . . . . . . 103 Gr aco . . . . . . . . . . . . . . . . . . . . . . . . . . . . . . . . . . . . . . . 104 Crescimento e Decrescimento da Fun ca o Am . . . . . . . . . . . . . . . . . 108 Estudo do Sinal e Inequa co es . . . . . . . . . . . . . . . . . . . . . . . . . . 109

10 Fun co es Quadr aticas

112

10.1 Deni co es e Exemplos . . . . . . . . . . . . . . . . . . . . . . . . . . . . . . 112 10.2 Forma Can onica e Zeros . . . . . . . . . . . . . . . . . . . . . . . . . . . . . 117 10.3 Valor M aximio ou M nimo e V ertice 10.4 Conjunto Imagem . . . . . . . . . . . . . . . . . . . . . . 120

. . . . . . . . . . . . . . . . . . . . . . . . . . . . . . . . 122

10.5 Eixo de Simetria da Par abola . . . . . . . . . . . . . . . . . . . . . . . . . . 124 10.6 Gr aco . . . . . . . . . . . . . . . . . . . . . . . . . . . . . . . . . . . . . . . 124 10.7 Estudo do Sinal e Inequa co es . . . . . . . . . . . . . . . . . . . . . . . . . . 126 11 Fun c ao Modular 129

11.1 M odulo ou Valor Absoluto de um N umero Real . . . . . . . . . . . . . . . . 130 11.2 Proprieades do M odulo de um N umero Real . . . . . . . . . . . . . . . . . . 130 11.3 Fun ca o Modular . . . . . . . . . . . . . . . . . . . . . . . . . . . . . . . . . . 131 11.4 Equa co es e Inequa co es Modulares . . . . . . . . . . . . . . . . . . . . . . . . 134

12 Fun co es Exponenciais

138

12.1 Pot encia de expoente inteiro . . . . . . . . . . . . . . . . . . . . . . . . . . . 138 12.2 Pot encia de expoente racional . . . . . . . . . . . . . . . . . . . . . . . . . . 139 12.3 Pot encia de expoente irracional . . . . . . . . . . . . . . . . . . . . . . . . . 141 12.4 Fun co es exponenciais . . . . . . . . . . . . . . . . . . . . . . . . . . . . . . . 142 12.5 Equa co es exponenciais . . . . . . . . . . . . . . . . . . . . . . . . . . . . . . 144 12.6 O n umero e . . . . . . . . . . . . . . . . . . . . . . . . . . . . . . . . . . . . 145 12.7 Inequa co es exponenciais . . . . . . . . . . . . . . . . . . . . . . . . . . . . . 146 13 Fun c ao Logar tmica 148

13.1 O logaritmo de um n umero . . . . . . . . . . . . . . . . . . . . . . . . . . . . 148 13.2 Algumas conseq u encias importantes da deni ca o de logaritmo . . . . . . . . 149 13.3 Propriedades operat orias do logaritmo . . . . . . . . . . . . . . . . . . . . . 149 13.4 Fun ca o logar tmica . . . . . . . . . . . . . . . . . . . . . . . . . . . . . . . . 150

13.5 Propriedades: . . . . . . . . . . . . . . . . . . . . . . . . . . . . . . . . . . . 151 13.6 Gr acos . . . . . . . . . . . . . . . . . . . . . . . . . . . . . . . . . . . . . . 152 13.7 Logaritmos decimal e natural . . . . . . . . . . . . . . . . . . . . . . . . . . 153

Cap tulo 1 Equa co es do 1 grau a uma inc ognita


Um passo decisivo na evolu ca o da matem atica foi a percep ca o de que a solu ca o de muitos problemas poderia car mais f acil e mais clara se em lugar da linguagem vernacular fosse usada uma linguagem simb olica. Vamos iniciar esta aula aprendendo, na se ca o 1.1, a escrever em linguagem simb olica os enunciados de alguns problemas. Logo depois, nas se co es seguintes, passaremos a abordar os m etodos de resolu ca o de equa co es.

1.1

Aprendendo uma linguagem simb olica

Situa c ao-problema Escreva as frases seguintes em linguagem simb olica: a) Um n umero acrescido de 42 e igual a 103; b) o triplo de um determinado n umero menos o dobro desse n umero e igual a 80; c) A metade de um determinado n umero mais um ter co desse n umero e igual a 116. Solu c ao: a) Represente por x o n umero mencionado no enunciado. enunciado do item (a) na forma seguinte: x + 42 = 103. 6 Podemos reescrever o

b) Represente por x o n umero mencionado nesse enunciado. Podemos reescrev e-lo na forma: 3x 2x = 80, onde 3x signica o triplo de x (isto e, 3 vezes x) e 2x signica o dobro de x (isto e, 2 vezes x). c) Represente por x o n umero mencionado nesta frase. Ent ao, podemos reescrev e-la da seguinte maneira: x x + = 116, 2 3 onde
x 2

designa a metade do n umero x e

x 3

designa a ter ca parte de x.

Atividade-proposta 1.1 Escreva as frases seguintes em linguagem simb olica: a) o triplo de um determinado n umero e igual ao seu dobro mais 4; b) um determinado n umero menos 17 e igual ao dobro desse n umero mais 4. c) A quarta parte de um n umero mais a sua quinta parte e igual a 115.

1.2

O m etodo da Inspe c ao Direta (ou das Tentativas)

De um modo geral, as equa co es a uma inc ognita aparecem associadas a alguns problemas nos quais se deseja determinar uma certa grandeza desconhecida (ou seja, uma inc ognita). As situa co es-problema seguintes exemplicam essa nossa arma ca o. O m etodo da Inspe ca o Direta ou das Tentativas ser a apresentado em suas solu co es. Situa c ao-problema Jo ao tinha uma certa quantia em sua conta banc aria. Ap os um dep osito de R$ 412,00 reais seu saldo banc ario passou para R$ 2.311,00. Quanto Jo ao tinha inicialmente, antes do dep osito, em sua conta banc aria?

Solu c ao: 7

Embora esse problema possa ser resolvido sem aux lio de uma equa c ao, n os a utilizaremos para ilustrar como ele pode ser escrito simbolicamente na forma de uma equa c ao. Represente por x a quantia que Jo ao tinha em sua conta banc aria antes do dep osito. Como diz o enunciado do problema, ap os um acr escimo de R$ 412,00, seu saldo passou para R$ 2.311,00. Ent ao podemos escrever: x + 412, 00 = 2.311, 00. Desse modo, a solu c ao do problema ca reduzida a descobrir-se qual e o n umero x que torna verdadeira a frase simb olica x + 412, 00 = 2.311, 00. Uma das maneiras de descobrir esse x e atrav es do m etodo da inspe c ao direta (ou das tentativas): vai-se atribuindo valores a x e testando qual deles funciona(ou seja, torna verdadeira a frase simb olica acima). Se voc e substituir x pelo n umero 1.800,00, a igualdade acima n ao se verica: 1.800, 00 + 412, 00 = 2.212, 00 e 2.212, 00 = 2.311, 00; observe que o valor obtido e menor do que 2.311,00. Se voc e substituir x por 1.900,00, por exemplo, a igualdade acima tamb em n ao se verica:

1.900, 00 + 412, 00 = 2.312, 00 e 2.312, 00 = 2.311, 00; observe agora que o valor obtido foi maior do que 2.311,00, diferindo deste por uma unidade. Portanto, se voc e substituir x por 1.899,00, a igualdade acima vai se vericar: 1.899, 00 + 412, 00 = 2.311, 00. Conclui-se, ent ao, que x = 1899, 00 e a solu c ao da equa c ao x + 412, 00 = 2.311, 00. 8

Portanto, Jo ao dispunha inicialmente de R$ 1.899,00 reais em sua conta banc aria. Situa c ao-problema Subtraindo-se 32 unidades de uma certa quantidade de objetos obtemos 11. Que quantidade de objetos e essa? Solu c ao: Represente por x a quantidade inicial de objetos. Pelo enunciado do problema, sabemos que x 32 = 11. Pelo m etodo da inspe c ao direta, e f acil concluir que a solu c ao dessa equa c ao e x = 43. Situa c ao-problema Sabe-se que o dobro de um n umero mais o triplo desse n umero e igual a 80. Que n umero e esse? Solu c ao Represente o n umero mencionado no enunciado por x. problema podemos concluir que 2x + 3x = 80. Resolveremos esta equa c ao pelo m etodo das tentativas. Experimente x = 20: 2 20 + 3 20 = 40 + 60 = 100. Ent ao, x = 20 n ao e solu c ao da equa c ao acima. Experimente, agora, x = 18 : 2 18 + 3 18 = 36 + 54 = 90. Logo, x = 18 tamb em n ao e solu c ao. Experimente, agora, x = 16 : 2 16 + 3 16 = 32 + 48 = 80. Portanto x = 16 e solu c ao da equa c ao acima. 9 Ent ao, pelo enunciado do

Atividade-proposta 1.2 Sabe-se que a metade de um n umero mais nove e igual ao dobro desse n umero. Que n umero e esse?

1.3

O M etodo Operat orio

Vimos na se ca o anterior uma t ecnica de resolu ca o de equa co es: o m etodo das tentativas (ou da inspe ca o direta). Passemos, agora, a abordar um outro m etodo de resolu ca o de equa co es chamado m etodo operat orio. Para apresentar esse m etodo, come caremos relembrando tr es propriedades

Aritm eticas dos n umeros reais: a) a exist encia de inversos multiplicativo e aditivo; b) as propriedades associativas da adi ca o e da multiplica ca o; c) os princ pios aditivo e multiplicativo.

1.3.1

A exist encia de inversos

A propriedade da exist encia do inverso multiplicativo pode ser enunciada da seguinte maneira: A cada n umero real a diferente de zero corresponde um inverso multiplicativo, isto e, um n umero a para o qual a a = a a = 1.
1 , porque Exemplo: O inverso multiplicativo de a = 2 e a = 2

a a =

1 1 2 = 1 e a a = 2 = 1. 2 2

A propriedade da exist encia do inverso aditivo pode ser enunciada da seguinte maneira: A cada n umero real a corresponde um inverso aditivo, isto e, um n umero a para o qual a + a = a + a = 0. 10

Exemplo: O inverso aditivo de a = 3 e o n umero a = 3 porque a + a = 3 + (3) = 0 e a + a = 3 + 3 = 0. Observa c ao: Note que estamos admitindo a exist encia do 0 (isto e, o elemento neutro da adi ca o) e do 1 (isto e, o elemento neutro da multiplica ca o). Atividade-proposta 1.3 Determine os inversos aditivo e multiplicativo dos n umeros 7, 108, 5 e 21.

1.4

As propriedades associativas da adi c ao e da multiplica c ao.

Propriedade associativas: Valem para quaisquer n umeros reais a, b e c as seguintes igualdades: a + (b + c) = (a + b) + c e a (b c) = (a b) c. A primeira igualdade e chamada propriedade associativa da adi c ao e a segunda, propriedade associativa da multiplica c ao. Exemplo Verique as igualdades seguintes: a) 5 + (7 + 9) = (5 + 7) + 9, b) 2 (3 5) = (2 3) 5. Solu c ao

11

a) Basta observar que 5 + (7 + 9) = 5 + 16 = 21 e (5 + 7) + 9 = 12 + 9 = 21. b) Basta observar que 2 (3 5) = 2 15 = 30 e (2 3) 5 = 6 5 = 30.

1.4.1

Os princ pios aditivo e multiplicativo

O princ pio aditivo da Aritm etica arma que se a = b, ent ao a + c = b + c, quaisquer que sejam os n umeros reais a, b e c. Ou seja, se adicionarmos uma mesma

constante a duas quantidades iguais obteremos ainda duas quantidades iguais. Exemplo: Se 2 + 3 = 3 + 2. ent ao (2 + 3) + 6 = (3 + 2) + 6. O princ pio multiplicativo da Aritm etica arma que se a = b, ent ao c a = c b, quaisquer que sejam os n umeros reais a, b e c. Exemplo: Se a = b, ent ao 2 a = 2 b, ou ainda 3 a = 3 b. O m etodo operat orio consiste em aplicar esses dois princ pios da Aritm etica e as propriedades operat orias (da associatividade, da distributividade e da exist encia de inversos e da exist encia de elementos neutros) a uma equa ca o dada a m de explicitar o valor de x. 12

Situa c ao-problema Resolva a equa c ao 3x = 1044, pelo m etodo operativo. Solu c ao: Precisamos determinar o valor de x para o qual esta igualdade e verdadeira. Para isso, multiplique ambos os membros dessa igualdade pelo inverso multiplicativo de 3, ent ao, pelo princ pio multiplicativo, obtemos 1 1 (3x) = 1044. 3 3 Pela propriedade associativa do produto, podemos escrever: 1 1044 3 x= . 3 3 Ent ao 1 x = 348. Logo, x = 348. Atividade-proposta 1.4 Resolva a equa ca o 5x = 350 pelo m etodo operat orio.

Atividade-proposta 1.5 Resolva a equa ca o x + 12 = 304 pelo m etodo operat orio.

Situa c ao-problema Mostre que se x + b = c, ent ao x = c b, para quaisquer n umeros reais c e b. Solu c ao: Adicione o inverso aditivo de b a ambos os membros da igualdade. Ent ao, pelo princ pio aditivo, obtemos (x + b) + (b) = c + (b) . 13

Pela associatividade dos n umeros reais, obtemos x + [b + (b)] = c b. Logo, x + 0 = c b. Donde x = c b. Situa c ao-problema Seja a um n umero real diferente de zero. Mostre que se ax = c, com a = 0, ent ao c x= . a

Solu c ao: Multiplique ambos os membros da equa c ao pelo inverso multiplicativo de a, para obter, pelo princ pio multiplicativo, a igualdade seguinte: 1 1 (ax) = c. a a Pela associatividade do produto, obtemos 1 c a x= . a a Logo, c x= . a Situa c ao-problema Resolva a equa ca o x + 7 = 121 pelo m etodo operat orio. Solu c ao: Adicione o inverso aditivo de 7 a ambos os membros dessa equa c ao. Ent ao, pelo princ pio aditivo, obtemos x + 7 + (7) = 121 + (7) . 14

Pela associatividade da adi c ao, teremos x + [7 + (7)] = 114. Logo, x + 0 = 114. Donde, x = 144. Atividade-proposta 1.6 A equa ca o geral do 1o grau a uma inc ognita e a equa ca o ax + b = 0, onde a,e b s ao n umeros reais, com a = 0. Mostre que a solu ca o desta equa ca o e dada por b x= . a

15

Cap tulo 2 Equa co es do 1 Grau a Uma Inc ognita


Nosso objetivo principal nesta aula e mostrar, na se ca o 2.1, como a utiliza ca o de inc ognitas pode ser utilizada na resolu ca o de alguns problemas, e conceituar, na se ca o 2.2, solu ca o de uma equa ca o a uma inc ognita.

2.1

Resolu c ao de alguns problemas

Situa c ao-problema: O dobro da idade de Jo ao adicionado ao seu triplo e igual a 85. Qual a idade de Jo ao? Solu c ao: Representaremos por x a idade de Jo ao. Pelo enunciado do problema sabemos que 3x + 2x = 85. A express ao 3x + 2x, no lado esquerdo da igualdade, e chamada primeiro membro da equa ca o e o n umero 85, no lado direito da igualdade, e chamado segundo membro da equa ca o. Podemos resolver essa equa c ao pelo m etodo das tentativas ou pelo m etodo operat orio. Optamos aqui pelo m etodo operat orio. 15

Note que adicionando 2x a 3x obtemos no primeiro membro da equa c ao o n umero 5x. Ent ao podemos reescrever essa equa c ao na forma 5x = 85. Multiplicando ambos os membros dessa equa c ao por 1/5 obtemos a seguinte igualdade: x= Donde, conclu mos que x = 17. Atividade-proposta 2.1 A soma do dobro da idade de Pedro e do triplo da idade de Maria e igual a 132 anos. Sabendo-se que a idade de Maria e um ter co da idade de Pedro, determine as idades de Pedro e de Maria. 85 . 5

Situa c ao-problema Tr es irm aos nasceram em anos consecutivos. Sabendo que a soma de suas idades e 159, qual a idade de cada um deles? Solu c ao: Suponha que a idade do mais novo e x. Ent ao as idades dos outros dois s ao x + 1 e x + 2. Pelo enunciado do problema, sabemos que x + (x + 1) + (x + 2) = 159. Efetuando a soma dos termos do primeiro membro dessa equa ca o, obtemos: 3x + 3 = 159. Adicionando 3 a ambos os membros dessa igualdade obtemos 3x = 156. 16

Multiplicando cada membro dessa u ltima equa c ao por 1/3, conclu mos x= 156 . 3

Logo x = 52. Portanto, as idades dos tr es irm aos ser ao 52, 53 e 54.

Atividade-proposta 2.2 Achar quatro n umeros consecutivos cuja soma e 174. Situa c ao-problema Resolva a equa ca o 3x + 4 = 2x 3. Solu c ao: Adicionando-se 2x a ambos os membros dessa equa c ao, obtemos, pelo Princ pio Aditivo, 2x + (3x + 4) = 2x + (2x 3) . Pela propriedade associativa, podemos reescrever (2.1) na forma (2x + 3x) + 4 = (2x + 2x) 3. Donde segue-se que x + 4 = 3. Adicionando 4 a ambos os membros dessa equa c ao, obtemos x = 7. Atividade-proposta 2.3 Resolva a equa ca o 2x + 5 = 4x 3, pelo m etodo operat orio. (2.1)

17

Situa c ao-problema Resolva a equa ca o 2 (3x + 8) = 3 (x + 1) .

Solu c ao: Usaremos na resolu c ao deste problema a propriedade distributiva dos n umeros reais, a qual arma que a igualdade a (b + c) = ab + ac, vale para todos os n umeros reais a, b e c. Desse modo, por essa propriedade de distributividade, podemos reescrever o primeiro membro da equa c ao na forma 2 (3x + 8) = 2 3x + 2 8 = 6x + 16, e o segundo membro na forma 3 (x + 1) = 3x + 3 1 = 3x + 3. Ent ao, a equa c ao dada assume a forma 6x + 16 = 3x + 3. Donde, pelo m etodo operat orio j a descrito anteriormente, chegamos ` a seguinte equa c ao: 6x 3x = 3 16. Conclu mos, ent ao, que 3x = 13. Finalmente, obtemos x= que e a solu c ao da equa c ao dada. 13 , 3

18

Atividade-proposta 2.4 Resolva a equa ca o 6 (x + 2) = 4 (3x + 1) , pelo m etodo operat orio. Situa c ao-problema Jo ao viveu a metade de sua vida em Macei o, quando mudou-se para S ao Paulo. Em S ao Paulo, faleceu ap os 23 anos. Quantos anos Jo ao viveu? Solu c ao: Seja x o n umero de anos vividos por Jo ao. Ent ao, pelo enunciado, temos a igualdade: x + 23 = x. 2 Multiplicando ambos os membros dessa equa c ao por 2, obtemos x + 46 = 2x. Logo, adicionando-se x a ambos os membros dessa igualdade, obtemos x = 46. Situa c ao-problema Resolva a equa ca o x x + 11 = . 3 4

Solu c ao: Multiplicando ambos os membros dessa equa c ao por 12, que e o m nimo m ultiplo comum dos denominadores 3 e 4, obtemos que 12 x x + 11 = 12 . 3 4

Donde, pela distributividade, conclu mos que 4x + 132 = 3x.

19

Adicionando 3x a ambos os membros dessa equa c ao, obtemos x + 132 = 0. Adicionando-se 132 a ambos os membros dessa u ltima equa c ao, conclu mos que x = 132. Atividade-proposta 2.5 Ache a solu ca o da equa ca o x x + 13 = , 4 8 pelo m etodo operat orio. Situa c ao-problema Um expresso gasta 4 horas, em m edia, de Macei o a Recife, enquanto um pinga-pinga gasta 6 horas, em m edia. Se o pinga-pinga partiu a `s 9 horas e o expresso partiu a `s 10 horas, a que horas o expresso alcan car a o pinga-pinga? Solu c ao: Observe que as velocidades m edias do expresso e do pinga-pinga s ao, respectivamente, ve = d d e vpp = , 4 6

onde d representa a dist ancia que os onibus percorrer ao de Macei o a Recife. Suponha, agora, que os dois onibus se encontram num momento t, depois do pinga-pinga partir. Como o pinga-pinga viajou t horas e o expresso viajou t 1 horas, ent ao os espa cos percorridos pelo expresso e pelo pinga-pinga s ao, respectivamente; spp = vpp t = se = ve (t 1) = Desse modo, no momento do encontro, temos que spp = se , 20 d t, 6 d (t 1) . 4

donde d d t = (t 1) . 6 4 Ent ao, multiplicando ambos os membros por


1 , d

obtemos

t t1 = . 6 4 Multiplicando por 24 ambos os membros, obtemos 4t = 6 (t 1) donde t = 3. Logo, os onibus se encontrar ao ` as 12 horas.

2.2

O que e resolver uma equa c ao?

Considere, por exemplo, a equa ca o x 4 = 1. Pergunta-se: o que e resolver essa equa ca o? Resolver essa equa ca o e determinar se algum elemento de um conjunto num erico dado, quando colocado no lugar de x, torna verdadeira a frase simb olica x 4 = 1. Situa c ao-problema Resolva a equa ca o x 4 = 1 no conjunto dos n umeros naturais N Solu c ao: Voc e vai encontrar facilmente, tanto pelo m etodo das tentativas quanto pelo m etodo operat orio, que x = 5 eou nico n umero que torna verdadeira a frase simb olica x 4 = 1. Portanto, x = 5 e a solu c ao da equa c ao dada. O conjunto N, no qual procuramos a solu c ao da equa c ao dada, e denominado conjunto universo da equa c ao. O elemento 5 do conjunto N que torna verdadeira a equa c ao dada e denominado solu c ao dessa equa c ao no conjunto universo N. 21

Temos, ent ao, em suma: Equa c ao dada: x 4 = 1 Conjunto universo: N Solu c ao: x = 5. Conjunto solu c ao: {5} Situa c ao-problema Considere agora a equa ca o x + 4 = 0. Resolva essa equa ca o: a) tendo o conjunto N dos n umeros naturais como conjunto universo. b) tendo o conjunto Z dos n umeros inteiros como conjunto universo; Solu c ao: f a) E acil se convencer atrav es do m etodo das tentativas de que em N a equa c ao dada n ao admite solu c ao. Logo, o conjunto solu c ao dessa equa c ao em N e o conjunto vazio. b) Em Z, a equa c ao dada admite uma solu c ao: x = 4 . Logo, o conjunto solu c ao

dessa equa c ao em Z e o conjunto {4}; Observa c ao: Essa situa c ao-problema acima mostra que a solu c ao de uma equa c ao depende do conjunto universo que estejamos considerando. Atividade proposta 2.6 1) Resolva a equa ca o x + 2 = 0: a) tendo N como conjunto universo; b) tendo Z como conjunto universo; c) tendo Q como conjunto universo. Situa c ao-problema Resolva a equa ca o 2x + 3 = 0: a) tendo o conjunto Q dos n umeros racionais como conjunto universo; b) tendo o conjunto N dos n umeros naturais como conjunto universo. Solu c ao: 22

f a) E acil vericar, pelo m etodo operat orio, que e solu ca o em Q da equa c ao dada. f b) E acil vericar, pelo m etodo das tentativas, que essa equa c ao n ao tem solu c ao no conjunto universo N. Atividade proposta 2.7 2) Resolva a equa ca o 2x 7 = 0: a) tendo N como conjunto universo; b) tendo Z como conjunto universo; c) tendo Q como conjunto universo. Situa c ao-problema Considere agora a equa ca o x2 4 = 0. Resolva essa equa ca o: a) tendo Z como conjunto universo; b) tendo N como conjunto universo; c) tendo A = {1, 3, 5} como conjunto universo. Solu c ao: a) Em Z, a equa c ao dada admite duas solu c oes: x = 2 e x = 2. Em Z, o conjunto solu ca o da equa c ao dada e o conjunto {2, 2}. b) Em N, a equa c ao dada admite uma u nica solu c ao: x = 2. Em N, o conjunto solu c ao dessa equa c ao e {2 }. c) No conjunto A = {1, 3, 5}, a equa c ao dada n ao admite solu c ao. Atividade-proposta 2.8 Considere agora a equa ca o x2 9 = 0. Resolva essa equa ca o: a) tendo Z como conjunto universo; b) tendo N como conjunto universo; c) tendo A = {1, 3, 5} como conjunto universo.

23

Cap tulo 3 Produtos Not aveis


Nosso principal objetivo nesta aula e investigar as propriedades de produtos do seguinte tipo: (a + b) (c + d) , onde a, b, c e d s ao n umeros reais quaisquer. Veremos que alguns produtos desse tipo apresentam algumas regularidades (isto e, algumas propriedades que se repetem) e cujo aprendizado permite um c alculo mais r apido ou mais direto dessas express oes. Tais produtos que apresentam essas regularidades s ao chamados de produtos not aveis. Situa c ao-problema Mostre que se a, b, c e d s ao n umeros reais quaisquer, ent ao (a + b) (c + d) = ac + ad + bc + bd.

Solu c ao: Podemos mostrar essa igualdade atrav es da aplica c ao da propriedade distributiva dos n umeros reais. Para ver isso, observe que a soma de dois n umeros reais e ainda um n umero real. Ent ao, podemos interpretar o fator (a + b) como um n umero real. Desse modo, pela

23

propriedade de distributividade dos n umeros reais, podemos escrever (a + b) (c + d) = (a + b) c + (a + b) d. Aplicando novamente a distributividade dos n umeros reais, obtemos (a + b) c = ac + bc e (a + b) d = ad + bd. Logo, conclu mos que (a + b) (c + d) = ac + bc + ad + bd. Observa c ao: Quando a, b, c e d s ao n umeros reais positivos, o produto (a + b) (c + d) pode ser interpretado como a rela ca o entre a a rea do ret angulo de lados (a + b) e (c + d) e as a reas dos ret angulos nele contidos:
ad bd d

ac

bc

3.1

O quadrado de uma soma

Situa c ao-problema Mostre que se a e b s ao n umeros reais, ent ao (a + b)2 = a2 + 2ab + b2 .

24

Solu c ao: Note que (a + b)2 = (a + b) (a + b) . Pelo mesmo argumento utilizado na situa c ao-problema anterior, obtemos as seguintes igualdades: (a + b) (a + b) = (a + b) a + (a + b) b = a2 + ab + ba + b2 = a2 + 2ab + b2 . Portanto, (a + b)2 = a2 + 2ab + b2 . Observa c ao: A igualdade (a + b)2 = a2 + 2ab + b2 e usualmente enunciada do seguinte modo: O quadrado de uma soma de dois termos e igual ao quadrado do primeiro termo mais duas vezes o produto do primeiro pelo segundo mais o quadrado do segundo. Observa c ao: Quando a e b s ao n umeros reais positivos o produto not avel (a + b)2 = a2 + 2ab + b2 pode ser interpretado como a rela ca o entre a a rea do quadrado de lados a + b e as a reas dos ret angulos nele contido, indicados na seguinte gura: Situa c ao-problema Calcule o seguinte produto (4x + y )2 , onde x e y s ao n umeros reais quaisquer. Solu c ao:

25

ab

b2

a2

ab

Pelo que vimos da situa c ao problema-anterior, vale a igualdade seguinte: (4x + y )2 = (4x)2 + 2 (4x) (y ) + y 2 = 16x2 + 8xy + y 2 .

Atividade proposta 3.1 a) Calcule os produtos (2m + 3n)2 e (a2 + 3b) . b) Dois inteiros positivos e consecutivos s ao tais que a diferen ca de seus quadrados e 65. Determine esses dois n umeros.
2

3.2

O quadrado de uma diferen ca

Situa c ao-problema Mostre que (a b)2 = a2 2ab + b2 , para quaisquer n umeros reais a e b. Solu c ao: Para obter este resultado, basta observar que a diferen ca de dois n umeros pode ser vista como uma soma. Para ver isso, basta notar que a b = a + (b) . 26

Desse modo, reca mos no caso anterior, podendo aplicar assim a f ormula obtida na situa c aoproblema anterior: (a b)2 = [a + (b)]2 = a2 + 2a (b) + b2 = a2 2ab + b2 . Observa c ao: Quando a e maior do que b a igualdade (a b)2 = a2 2ab + b2 e usualmente enunciada do seguinte modo: o quadrado da diferen ca de dois termos e igual ao quadrado do primeiro menos duas vezes o produto do primeiro pelo segundo mais o quadrado do segundo. Observa c ao: Quando a e maior do que b a igualdade (a b)2 = a2 2ab + b2 pode ser intepretada geometricamente por interm edio da gura abaixo:

b(a-b)

b2

a (a-b)2 b(a-b) a-b

a-b a

Nessa gura, verica-se a seguinte igualdade:

a2 = (a b)2 + 2b (a b) b2 . 27

Da , conclu mos que (a b)2 = a2 2b (a b) b2 . Logo, (a b)2 = a2 2ab + b2 . Situa c ao-problema Calcule o produto (3m 2n)2 . Solu c ao: Pela f ormula vista na situa c ao-problema anterior, (3m 2n)2 = (3m)2 2 (3m) (2n) + (2n)2 = 3m2 12mn + 4n2 . Atividade proposta 3.2 Calcule os produtos (2x 4y )2 e (a 3b)2 .

3.3

A diferen ca de quadrados

Situa c ao-problema Mostre que a2 b2 = (a + b) (a b) . para quaisquer n umeros reais a e b. Solu c ao: Para ver isso, observe que (a + b) (a b) = (a + b) [a + (b)] .

28

Aplique agora a propriedade distributiva dos n umeros reais para obter (a + b) [a + (b)] = (a + b) a + (a + b) (b) = a2 + ba + a (b) + b (b) = a2 + ba ab b2 = a2 b2 . Logo, (a + b) (a b) = a2 b2 . Situa c ao-problema Determine o produto (2m + 5n) (2m 5n) . Solu c ao: Pela f ormula vista na situa c ao-problema anterior, (2m + 5n) (2m 5n) = (2m)2 (5n)2 = 4m2 25n2 . Atividade proposta 3.3 Determine o produto (3x + 2y ) (3x 2y ) . Atividade proposta 3.4 Mostre que (x + a) (x + b) = x2 + (a + b) x + ab. Al em disso, d e uma interpreta ca o geom etrica a essa f ormula. Atividade proposta 3.5 Use a f ormula dada na atividade proposta anterior para calcular: a) (x + 2) (x + 3) b) (x + 2) (x 3) c) (x 2) (x 3) . 29

3.4

O cubo de uma soma

Situa c ao-problema Mostre que (a + b)3 = a3 + 3a2 b + 3ab2 + b3 , quaisquer que sejam os n umeros reais a e b. Solu c ao: Para ver isso, basta observar que (a + b)3 = (a + b) (a + b)2 = (a + b) a2 + 2ab + b2 . Pela propriedade distributiva dos n umeros reais, obtemos (a + b) a2 + 2ab + b2 = a a2 + 2ab + b2 + b a2 + 2ab + b2 = a3 + 2a2 b + ab2 + ba2 + 2ab2 + b3 = a3 + 3a2 b + 3ab2 + b3 . Atividade proposta 3.6 Calcule: a) (a + 2)3 b) (2x + 3y )3 . Atividade proposta 3.7 Mostre que (a b)3 = a3 3a2 b + 3ab2 b3 , quaisquer que sejam os n umeros reais a e b.

30

Atividade proposta 3.8 D e uma interpreta ca o geom etrica para a f ormula (a + b)3 = a3 + 3a2 b + 3ab2 + b3 , para o caso em que a e b s ao n umeros reais positivos.

31

Cap tulo 4 Fatora c ao


Agora vamos estudar uma opera ca o fundamental em Algebra: a fatora ca o. Certamente voc e j a ouviu falar no Ensino Fundamental sobre a fatora ca o de um n umero. Vamos iniciar nossa aula sobre fatora ca o recordando alguns fatos b asicos acerca da fatora ca o de um n umero.

4.1

Fatora c ao de um n umero

Como voc e deve se lembrar, fatorar um n umero natural e escrev e-lo como um produto de n umeros naturais. Desse modo, todo n umero natural n admite duas fatora co es triviais, as quais consistem em escrev e-lo nas formas n = n 1 e n = 1 n. Salientamos que neste texto o conjunto dos n umeros naturais e o conjunto N = {1, 2, 3, . . .} dos inteiros positivos. Exemplo:

31

O n umero 12 pode ser fatorado de v arias maneiras: 12 = 12 1 12 = 1 12 12 = 2 6 12 = 3 4 12 = 4 3 12 = 2 2 3 12 = 3 2 2 12 = 2 3 2. Quando fatoramos o n umero 12, por exemplo, na forma 12 = 6 2, dizemos que 6 e 2 s ao os fatores do n umero 12. Atividade proposta 4.1 Fatore 18 e 50 de v arias maneiras.

4.2

Fatora c ao de um n umero em fatores primos

O nosso objetivo nessa se ca o e aprender (ou recordar) alguns fatos b asicos sobre a fatora ca o de um n umero em fatores primos. Para entender o que e um n umero primo, lembremos o conceito da divisibilidade de um n umero natural. Dizemos que um n umero natural a e divisivel por um n umero b diferente de zero quando podemos escrever a da seguinte maneira: a = b c, para algum n umero natural c. Neste caso, dizemos tamb em que b e divisor de a ou que a e um m ultiplo de b 32

Exemplo: 8 e divis vel por 2, porque 8 = 2 4; neste caso, dizemos tamb em que 2 e divisor de 8. 8 e divis vel por 4, porque 8 = 4 2; neste caso, dizemos que 4 e divisor de 8. 8 n ao e divisivel por 3, porque n ao existe nenhum n umero inteiro c para o qual 8 = 3 c. Atividade proposta 4.2 Responda, justicando: a) o n umero 3 e divisor de 21? b) o n umero 3 e divisor de 17? c) o n umero 17 e divis vel por 4? d) o n umero 24 e divis vel por 4?

Exemplo: Todo n umero natural n e divis vel por 1, porque sempre podemos escrever n = 1 n. Todo n umero natural n tamb em e divis vel por ele mesmo, porque sempre temos a igualdade seguinte: n = n 1. Situa c ao-problema: Determine todos os divisores de 12. Solu c ao: Os divisores de 12 s ao os n umeros 1,2,3,4,6 e 12, como voc e facilmente pode constatar. Atividade proposta 4.3 Determine todos os divisores do n umero 18, 21 e 32. Passemos, agora, ao conceito de n umero primo. Um n umero natural n e primo quando e maior do que 1 e seus u nicos divisores s ao os n umeros 1 e n. Exemplo: 33

Os n umeros 2,3,5 e 7 s ao n umeros primos. Agora, o n umero 6 n ao e um n umero primo, porque ele admite outros divisores al em de 1 e 6: 6= 23 6 = 3 2. Ou seja, 2 e 3 s ao divisores de 6. Atividade proposta 4.4 Responda a `s seguintes perguntas: a) 11 e um n umero primo? Justique. b) 15 e um n umero primo? Justique. Se voc e observar bem, vai notar que n umeros que n ao s ao primos podem ser escritos como um produto de n umeros primos; por exemplo, 4=22 6=23 8= 222 9 = 3 3. Pode-se mostrar que isso e sempre verdade, ou seja, que todo n umero inteiro maior do que 1 e um produto de n umeros primos. Esse fato e conhecido em Matem atica como o Teorema Fundamental da Aritm etica, cuja demonstra ca o ser a vista em um curso posterior sobre teoria dos n umeros. Em outras palavras, o Teorema Fundamental da Aritm etica arma que todo n umero inteiro n maior do que 1 que n ao e primo pode ser escrito como um produto de n umeros primos. Cada n umero primo que aparece nesse produto e chamado fator do n umero inteiro n. Situa c ao problema: Fatore 24 e 36 como um produto de fatores primos. 34

Solu c ao: 24 2 12 2 6 3 1 2 3 Ent ao podemos escrever 24 = 2 2 2 3 = 23 3.

36 2 18 2 9 3 1 3 3 Ent ao podemos escrever 36 = 2 2 3 3 = 22 32 .

4.3

Fatora c ao de Mon omios

Como veremos adiante neste curso, e comum encontrarmos somas do tipo x2 + 2x2 y + x + 3 cujas parcelas envolvem apenas inc ognitas (como e o caso de x2 e x), produtos de n umero e inc ognitas (como e o caso de 2 x2 y ) ou ainda n umeros isolados (como e o caso de 3). Essas express oes que envolvem apenas n umeros isolados ou inc ognitas isoladas, ou pot encias de inc ognitas, ou ainda produtos de n umero e pot encias de inc ognitas s ao chamadas mon omios. Situa c ao-problema Quais das express oes abaixo s ao mon omios? Justique. a) 2xy b) 2xy + x2 c) x + 2 d) 4a2 b. 35

Solu c ao: a) A express ao 2xy e um mon omio, porque e constitu da apenas por produtos de um n umero ( o 2) e inc ognitas (x e y). b) A express ao 2xy + x2 n ao e um mon omio, porque envolve uma adi c ao. c) A express ao x + 2 n ao e um mon omio, porque envolve uma soma. d) A express ao 4a2 b e um mon omio, porque e constitu do apenas por produtos de n umero e inc ognitas. Atividade proposta 4.5 Decida quais das express oes abaixo s ao mon omios, apresentando uma justicativa: a) x2 b) x2 + x3 c) a2 b3 d) 2. Passemos agora a ` fatora ca o de mon omios. Fatorar um mon omio e escrev e-lo como um produto de outros mon omios. Situa c ao-problema Fatore o mon omio 4x3 de v arias maneiras. Solu c ao:

4x3 = 4x x2 4x3 = 2x 2x2 4x3 = 4x2 x 4x3 = x 4x2 4x3 = x x2 4 4x3 = 4 x x x.

36

Atividade proposta 4.6 Fatore o mon omio 8x3 y de cinco maneiras distintas.

4.4

Fatora c ao de Bin omios

Um bin omio e uma soma de dois mon omios. Exemplo: As express oes x2 +2xy, x 2 e A2 B 2 s ao bin omios, porque s ao constitu dos pela adi c ao de dois mon omios:

x2 + 2xy e a adi ca o dos mon omios x2 e 2xy ; x2 e a adi ca o dos mon omios x e 2; A2 B 2 e a adi ca o dos mon omios A2 e B 2 . Atividade proposta 4.7 Decida quais das express oes abaixo s ao bin omios: a) x3 + 2x b) x2 y c) x2 + 2 d) x3 + 2x + y. Passemos, agora, a ` fatora ca o de bin omios. Fatorar um bin omio e escrev e-lo como um produto. O primeiro modo de fatorar um bin omio e pela t ecnica do fator comum, a qual passaremos a abordar na situa c ao-problema seguinte: Situa c ao-problema Fatore o bin omio 4a2 b + 8ab. Solu c ao: 37

A fatora c ao de um bin omio pela t ecnica do fator comum consiste em se destacar em cada uma de suas parcelas um fator comum investigando cada uma delas: 1a parcela: 4a2 b = 4 a a b 2a parcela: 8ab = 2 4 a b. Note que 4ab aparece em ambas as parcelas, constituindo-se assim em um fator comum. Ent ao, podemos colocar esse fator comum 4ab em evid encia fazendo uso da propriedade distributiva dos n umeros reais: 4a2 b + 8ab = 4ab a + 4ab 2 = 4ab (a + 2) . Atividade proposta 4.8 Fatore os bin omios abaixo colocando em evid encia um fator comum: a) 4x2 + 6x b) 10a3 5a2 c) 3x3 6xy d) 4x2 16. Passaremos, agora, a uma outra t ecnica de fatora ca o, a dos produtos not aveis, a qual se aplica a express oes do tipo, A2 B 2 , onde A e B s ao mon omios. Utilizando nossos conhecimentos j a aprendidos na aula sobre produtos not aveis, sabemos que tais bin omios podem ser escritos na forma A2 B 2 = (A B ) (A + B ) . Situa c ao problema Fatore o bin omio 4x2 9. Solu c ao: 38

Para fazer essa fatora c ao, basta abservar que 4x2 9 = (2x)2 32 . Ent ao, pela f ormula acima, podemos escrever 4x2 9 = (2x 3) (2x + 3) . Aqui A = 2x e B = 3. Atividade proposta 4.9 Fatore os bin omios abaixo: a) x2 4 b) 4a2 9b2

4.5

Fatora c ao de Trin omios

Um trin omio e uma soma de tr es mon omios. Por exemplo, x2 + 2x + 3 e a2 + b + 3c3 s ao trin omios mas x3 + 2x e x2 + 4ab + 2b + 6 n ao s ao trin omios. Abordaremos neste curso duas t ecnicas de fatora ca o de trin omios. A primeira e a t ecnica do fator comum e a segunda e a dos produtos not aveis. Come caremos pela t ecnica do fator comum. Situa c ao-problema Fatore o trin omio 2x3 + 4x2 + 6x. Solu c ao: Analogamente ao caso dos bin omios, podemos fatorar esse trin omio buscando destacar em cada uma de suas parcelas um termo comum:

primeira parcela: 2x3 = 2 x x x segunda parcela: 4x2 = 2 2 x x terceira parcela:6x = 2 3 x. 39

Note que 2x e um fator comum ` as tr es parcelas. Ent ao, podemos escrever 2x3 + 4x2 + 6x = 2x x2 + 2x 2x + 2x 3 = 2x x2 + 2x + 3 .

Atividade proposta 4.10 Fatore os trin omios abaixo pelo m etodo do fator comum: a) 8x4 6x3 + 4x2 b) 4x3 x2 + 2x. Passemos agora a ` t ecnica dos produtos not aveis. Essa t ecnica se baseia na utiliza ca o dos produtos not aveis (A + B )2 = A2 + 2AB + B 2 (A B )2 = A2 2AB + B 2 . Essas f ormulas dizem que os trin omios do tipo A2 + 2AB + B 2 ou A2 2AB + B 2 podem ser escritos como os produtos (A + B ) (A + B ) e (A B ) (A B ), respectivamente. Situa c ao-problema Decida para quais dos trin omios abaixo e v alida a f ormula A2 + 2AB + B 2 = (A + B )2 : a) x2 + 6x + 9 b) x2 + 6x + 6 c) 4x2 + 12x + 16. Solu c ao: v a) E alida neste caso, porque x2 + 6x + 9 = x2 + 2 3x + 32 logo, pela f ormula do produto not avel, obtemos x2 + 6x + 9 = (x + 3)2 . 40

Aqui A = x e B = 3. b) N ao e v alida. Porque x2 + 6x + 6 = x2 + 2 3 x + 32 . v c) E alida. Porque 4x2 + 16x + 16 = (2x)2 + 2 4 2x + 42 . E a f ormula se verica para A = 2x e B = 4. Ent ao, 4x2 + 16x + 16 = (2x + 4)2 .

Atividade proposta 4.11 Decida para quais dos trin omios abaixo a f ormula A2 + 2AB + B 2 = (A + B )2 e v alida: a) x2 + 8x + 16 b) 9x2 + 12x + 4 c) x2 + 8x + 5. Situa c ao-problema Decida para quais dos trin omios abaixo a f ormula A2 2AB + B 2 = (A B )2 e v alida: a) x2 6x + 9 b) x2 6x + 5. Solu c ao: a) x2 6x + 9 = x2 2 3x + 32 = (x 3)2 . b) N ao e v alida, porque x2 6x + 5 = x2 2 3x + 32 .

41

Atividade proposta 4.13 Fatore as seguintes express oes: a) x2 + 6x + 9 b) x2 4x + 4 c) x4 + 8x2 + 16 d) 4x2 12x + 9.

42

Cap tulo 5 Equa co es Quadr aticas


Nosso objetivo principal nesta aula e aprender a identicar e resolver uma equa ca o quadr atica reconhecendo as condi co es sobre as quais ela admite nenhuma, uma ou duas solu co es reais. Vamos iniciar esta aula aprendendo a identicar uma equa ca o quadr atica a uma inc ognita em sua forma geral. Em linguagem simb olica, a equa c ao quadr atica geral a uma inc ognita e a coecientes reais e uma express ao do tipo ax2 + bx + c = 0, na qual os coecientes a, b e c s ao n umeros reais previamente conhecidos, com a = 0, e x representa uma inc ognita (isto e, um n umero ou uma quantidade cujo valor se deseja determinar). Abaixo listamos alguns exemplos de equa co es quadr aticas: a) 2x2 3x + 1 = 0, (aqui a = 2, b = 3 e c = 1) b) x2 + 4 = 0 (aqui a = 1, b = 0 e c = 4);

42

Atividade proposta 5.1 Determine os coecientes (a, b e c) das equa co es quadr aticas abaixo: a) 3x2 + x + 3 = 0 b) x2 2x = 0 c) x2 4 = 0.

5.1

Equa c oes quadr aticas incompletas

Quando na equa ca o quadr atica geral temos b = 0 ou c = 0, dizemos que a equa ca o quadr atica e incompleta. Exemplo: As equa co es 2x2 + 4x = 0 e x2 + 6 = 0 s ao equa co es quadr aticas incompletas. Situa c ao-problema Determine a aresta de um quadrado cujo dobro da a rea e igual a 32 m2 . Solu c ao: Seja x a aresta do quadrado. Pelo enunciado do problema, sabemos que 2x2 = 32. Ent ao, x2 = 16. Donde, x = 4 ou x = 4. Portanto, a aresta do quadrado mede 4m. Situa c ao-problema Determine x, sabendo que 4x2 + 16x = 0.

Solu c ao: 43

Observe que 4x e um fator comum do bin omio que aparece no primeiro membro da equa ca o dada. Ent ao, fatorando o primeiro membro da equa c ao, obtemos 4x (x + 4) = 0. (5.1)

Para resolver esta equa ca o, utilizaremos o seguinte fato: se A e B s ao n umeros reais tais que A B = 0, ent ao A = 0 ou B = 0. (Veja a atividade proposta 5.3 adiante) Desse modo da igualdade (5.1) acima podemos concluir que 4x = 0 ou x + 4 = 0. Logo, temos duas solu c oes poss veis para o problema: x = 0 ou x = 4. Atividade-proposta 5.2 Determine x, sabendo que: a) x2 18x = 0 b) 4x2 + 8x = 0 Atividade proposta 5.3 Sejam A e B n umeros reais. Mostre que se A B = 0, ent ao A = 0 ou B = 0.

5.2

Equa c oes quadr aticas completas

Quando numa equa ca o quadr atica geral os coecientes a, b e c s ao todos diferentes de zero dizemos que ela e completa.

5.2.1

Casos em que o trin omio ax2 + bx + c e um quadrado perfeito.

Situa c ao-problema Resolva a equa ca o x2 + 4x + 4 = 0.

Solu c ao: 44

Note que o trin omio do primeiro membro da equa c ao pode ser fatorado por interm edio do produto not avel (A + B )2 = A2 + 2AB + B 2 . Basta observar que x2 + 4x + 4 = x2 + 2 2x + 22 = (x + 2)2 . Ent ao a equa c ao dada pode ser reescrita na forma (x + 2)2 = 0. O u nico n umero cujo quadrado e igual a zero e o pr oprio zero, logo x + 2 = 0. Donde, x = 2. Atividade proposta 5.4 Resolva a equa ca o x2 + 6x + 9 = 0, utilizando o produto not avel (A + B )2 = A2 + 2AB + B 2 Situa c ao-problema Resolva a equa ca o x2 8x + 16 = 0.

Solu c ao: Basta notar que x2 8x + 16 = x2 2 4x + 42 . Ent ao a equa c ao pode ser reescrita na forma (x 4)2 = 0. Logo, x 4 = 0. Donde, x = 4. 45

5.2.2

Casos em que o trin omio ax2 + bx + c n ao e um quadrado perfeito (completamento de quadrado)

Situa c ao-problema Determine x, sabendo que x2 + 6x + 5 = 0.

Solu c ao: Neste caso n ao podemos utilizar nenhum produto not avel, porque x2 + 6x + 5 = x2 + 2 3x + 32 . Embora o primeiro membro da equa c ao n ao seja um quadrado perfeito, podemos aproxim alode um quadrado perfeito escrevendo-o do seguinte modo: x2 + 6x + 5 = x2 + 2 3x + 5 = x2 + 2 3x + 32 32 + 5. Na primeira igualdade, somamos e subtra mos 32 , n ao alterando o valor do primeiro membro. Essa t ecnica e chamada completamento de quadrado. Pelas considera c oes acima, podemos reescrever a equa ca o dada na forma x2 + 2 3x + 32 9 + 5 = 0. Ent ao, pelo que vimos na aula sobre produtos not aveis, (x + 3)2 4 = 0. Donde, (x + 3)2 = 4. Logo, x + 3 = 2 ou x + 3 = 2. 46

Portanto, x = 1 ou x = 5. Atividade proposta 5.5 Resolva as equa co es usando completamento de quadrado: a) x2 + 8x + 1 = 0 b) x2 2x + 7 = 0. Situa c ao-problema Considere a equa ca o quadr atica x2 + Bx + C = 0, onde B e C s ao n umeros reais quaisquer. Mostre que se as solu co es dessa equa ca o existirem, ent ao elas ser ao dadas por B + B 2 4C B B 2 4C x1 = e x2 = . 2 2

Solu c ao: Por completamento de quadrados, obtemos B x + Bx + C = 0 = x + 2 x + 2


2 2

B 2

+C
2

B 2

=0

observe agora que B x +2 x+ 2


2

B 2

B x+ 2
2

donde obtemos x+ Da segue-se que x+ B 2


2

B 2

+C

B 2

= 0.

B C 2 B 2 4C = . 4 47

Logo, se B 2 4C 0, ent ao x+ Donde x=

B = 2 B

B 2 4C 4 B 2 4C . 2

Observa c ao: A resolu ca o da situa ca o-problema acima mostra que se B 2 4C 0, ent ao existe pelo menos uma solu ca o real para a equa ca o x2 + Bx + C = 0. Situa c ao-problema: Determine as solu co es da equa ca o x2 3x + 2 = 0, caso existam. Solu c ao: Usando as f ormulas dadas na situa c ao-problema anterior, e notando que B = 3 e C = 2, obtemos (3) + (3)2 4 (2) 3+ 1 = =2 2 3 1 = = 1. 2

x1 = e x2 =

(3)

(3)2 4 (2)

Portanto x1 = 2 e x2 = 1, s ao as solu co es da equa ca o dada. Situa c ao-problema Considere a equa ca o quadr atica ax2 + bx + c = 0, a = 0. Mostre que se as solu co es dessa equa ca o existirem elas ser ao dadas por b x= , 2a onde = b2 4ac. Solu c ao: 48

b x+ Basta observar que se ax 2 + bx + c = 0, ent ao a x2 + a

c a

= 0. Deste modo obtemos

b c x2 + x + = 0. a a
b c e C=a , obtemos Aplicando a situa c ao-problema anterior, com B = a b x= . 2a

Ou seja, a equa ca o quadr atica geral ax2 + bx + c = 0 tem no m aximo duas solu co es dadas por b b + x1 = e x2 = , 2a 2a onde = b2 4ac. Observa c ao 1: A express ao = b2 4ac e chamada de discriminante da equa ca o ax2 + bx + c = 0, a = 0. Observa c ao 2: O n umero de solu co es reais da equa ca o ax2 + bx + c = 0, a = 0 e dado pelo sinal do discriminante: a) Se > 0, essa equa ca o tem duas solu co es reais; b) Se = 0, essa equa ca o tem uma u nica solu ca o real; c) Se < 0, essa equa ca o n ao tem solu ca o real. Atividade proposta 5.6 a) A soma de dois n umeros e 24 e produto e 143. Determine esses n umeros b) Mostre que n ao existe n umeros inteiros consecutivos cujo produto e igual a 57. c) Um pai tinha 30 anos quando seu lho nasceu. Sabe-se que o produto da idade do pai e da idade do lho e igual ao quadrado da idade do lho. Quais s ao suas idades?

49

5.3

Fatora c ao de trin omios

Situa c ao-problema Mostre que se x1 e x2 s ao as duas solu co es de ax2 + bx + c = 0, ent ao a (x x1 ) (x x2 ) = ax2 + bx + c.

Solu c ao: Sejam x1 =


b+ 2a

e x2 =

b 2a

as duas solu co es da equa ca o ax2 + bx + c = 0.

Note inicialmente que (x x1 ) (x x2 ) = x2 (x1 + x2 ) x + x1 x2 . Mas b b b + + = . x1 + x2 = 2a 2a a b + b 1 x1 x2 = = 2 b2 2a 2a 4a 1 b2 b2 4ac 2 4a c = . a = Logo, b c (x x1 ) (x x2 ) = x2 x + . a a Portanto, a (x x1 ) (x x2 ) = ax2 + bx + c. Observa c ao: Essa u ltima situa ca o-problema fornece um novo procedimento de = 4ac 4a2

fatora ca o do trin omio ax2 + bx + c. Para obter uma fatora ca o desse trin omio, basta determinar duas solu co es x1 e x2 da equa ca o ax2 + bx + c = 0 e escrever ax2 + bx + c = a (x x1 ) (x x2 ) . 50

Note que se 0, ent ao o trin omio ax2 + bx + c pode ser fatorado e dizemos ent ao que ele e redut vel. Contudo se < 0 ent ao a equa ca o ax2 + bx + c = 0 n ao admite solu ca o real. Donde segue-se que o trin omio ax2 + bx + c n ao pode ser fatorado. Neste caso dizemos que ele e irredut vel no conjunto dos n umeros reais. Situa c ao-problema Fatore o trin omio 3x2 3x 6 = 0.

Solu c ao: Determinemos as solu co es da equa ca o 3x2 3x 6 = 0: (3) + b + x1 = = 2a (3) b = x2 = 2a Logo 3x2 3x 6 = 3 (x 2) (x + 1) . Atividade proposta 5.7 Fatore o trin omio 3x2 + 6x 9 Atividade proposta 5.8 Classique em redut vel ou irredut vel os trin omios abaixo: a) x2 2x + 3 b) x2 + 4 c) x2 2x 3 d) 2x2 + x 1 (3)2 4 3 (6) 23 = 3+ 6 6 81 =2

(3)2 4 3 (6) 23

81

= 1.

51

Cap tulo 6 Fra co es Alg ebricas


Objetivo: aprender a simplicar e adicionar fra co es alg ebricas.

6.1

Fra c oes num ericas

Antes de come carmos a trabalhar com as fra co es alg ebricas, vamos recordar alguns fatos b asicos acerca das fra co es num ericas. Como voc e ainda se lembra, uma fra c ao num erica e um quociente do tipo A (l e-se: A sobre B ou A dividido por B ), B onde A e B s ao n umeros reais, sendo B = 0. O n umero A e denominado numerador e o B e chamado denominador. Exemplo: os quocientes 1 , 3 Uma fra ca o num erica
A B 2 40 , 3 25

3 1

s ao fra co es num ericas.

e redut vel quando os n umeros A e B apresentam pelo menos

um divisor comum maior do que 1; quando A e B n ao apresentam pelo menos um divisor comum maior do que 1 diz-se que a fra ca o Exemplos: a) A fra ca o num erica
4 8 A B

e irredut vel.

e redut vel, porque 4 e 8 t em um divisor comum: o n umero 2. 51

b) A fra ca o 2/3 e irredut vel, porque o numerador e o denominador n ao possuem nenhum divisor comum maior do que 1. Atividade proposta 6.1 Decida quais das fra co es num ericas abaixo s ao redut veis, justicando sua resposta: 5 3 9 2 1 4 , , , , , . 25 27 1 5 8 50

Simplicar uma fra ca o num erica redut vel e dividir o seu numerador e o seu denominador por seus divisores comuns. A fra ca o irredut vel que se obt em ap os a simplica ca o de uma fra ca o redut vel chama-se forma simplicada da fra ca o redut vel. Situa c ao-problema Simplique as fra co es
36 30

25 . 45

Solu c ao: Fatorando numerador e o denominador da primeira fra ca o em seus fatores primos, obtemos 36 2233 = . 30 235

Dividindo o numerador e o denominador pelos fatores comuns, 2 e 3, obtemos 36 2 2 3 3 23 6 = = = . 3 2 3 5 5 5 A fra ca o


6 5

e a forma simplicada de

36 . 30

Para a segunda fra ca o, basta observar que 25 55 55 5 = = = . 45 59 59 9 A fra ca o irredut vel


5 9

e a forma simplicada de

25 . 45

Atividade proposta 6.2 Simplique as fra co es Uma fra ca o redut vel simplicada de
A B 16 48 A B

21 . 49 C D

e uma fra ca o irredut vel


C . D

s ao equivalentes quando a forma

e igual a

52

Duas fra co es redut veis Situa c ao-problema

A B

C D

s ao equivalentes quando t em a mesma forma simplicada.

Estude a equival encia das seguintes fra co es: a) b) c)


2 3 4 8 4 8 5 e 6 ;

e 1 ; 2 e
16 . 32

Solu c ao: a) Note que equivalentes. b) Note que


1 2 2 3

5 6

s ao fra co es irredut veis. Como

2 3

= 5 , ent ao essas fra co es n ao s ao 6

e irredut vel e que

4 8

e redut vel. Observe tamb em que

4 2 2 1 = = . 8 2 2 2 2 Como a forma simplicada de c) Note que as duas fra co es Observe que 4 22 1 = = 8 222 2 e 16 2222 1 = = . 32 22222 2 Logo, como as fra co es dadas possuem a mesma forma simplicada, elas s ao equivalentes. Atividade proposta 6.3 Estude a equival encia das seguintes fra co es: a) b) c)
2 3 2 3 30 45 7 e 8 ; 4 8 4 8

e igual a 1 , ent ao 2
16 32

4 8

1 2

s ao equivalentes.

s ao redut veis.

e e

20 ; 30 40 . 60

53

6.2

Fra c oes alg ebricas

Um polin omio e uma soma de mon omios. Exempo: x3 + 2x2 y e um polin omio, porque e uma soma de dois mon omios: x3 e 2x2 y. ao e um polin omio, porque 2 x n ao e um mon omio. Mas 2 x + 3 n Uma fra c ao alg ebrica e um quociente cujos numerador e denominador s ao polin omios (fatorados ou n ao). Exemplo: Os quocientes 5a (x + 1) (x 2) 7x2 + 3x , e 2 2 x + x 4ab + ba x2 2x + 4 s ao fra co es alg ebricas, porque s ao quocientes de polin omios. Uma fra ca o alg ebrica e redut vel quando o seu numerador e o seu denominador apresentam pelo menos um fator comum. Situa c ao-problema 3x3 + 4xy e redut vel? A fra ca o alg ebrica 2 x + 2xy Solu c ao: Para responder essa pergunta, fatore o numerador e o denominador dessa fra ca o, para vericar se eles possuem fator comum; numerador : 3x3 + 4xy = x (3x2 + 4y ) denominador : x2 + 2xy = x (x + 2y ) . Como o numerador e o denominador apresentam um fator comum (o x), conclu mos que a fra ca o dada e redut vel. Atividade proposta 6.4 a3 + ab a) A fra ca o 2 e redut vel? Justique. 2a + 5ab 2 x + 2xy b) A fra ca o e redut vel? Justique. 2y + y 2

54

6.3

Simplica c ao de fra c oes alg ebricas

Simplicar uma fra ca o alg ebrica redut vel e dividir o seu numerador e o seu denominador por todos os seus fatores comuns. Situa c ao-problema Simplicar as fra co es abaixo: 3 3x + 4xy a) 2 , x = 0; x + 2x (a + h)2 a2 b) , h = 0; h c) d) (x + 1) (x 2) , x = 0; x2 4x + 4 (x 3)2 , x = 2 e x = 3. x2 5x + 6

Solu c ao: a) Para simplicar a primeira fra ca o, fatora-se o seu numerador e o seu denominador, e em seguida divide-se o numerador e o denominador por seus fatores comuns: x (3x2 + 4y ) (3x2 + 4y ) 3x3 + 4xy = = . x2 + 2x x (x + 2) (x + 2) Paramos aqui, porque o numerador e o denominador dessa u ltima fra ca o n ao t em mais nenhum fator comum. b) Antes de fatorar o numerador, podemos efetuar algumas opera co es preliminares: (a + h)2 a2 a2 + 2ah + h2 a2 2ah + h2 = = . h h h Podemos, agora, fatorar essa u ltima fra ca o alg ebrica: 2ah + h2 h (2a + h) = = 2a + h. h h Portanto, (a + h)2 a2 = 2a + h, para h = 0. h c) O numerador j a est a fatorado. O denominador pode ser fatorado por interm edio de uma das f ormulas dos produtos not aveis que voc e j a sabe: 55

x2 4x + 4 = (x 2)2 = (x 2) (x 2) . Ent ao, (x + 1) (x 2) (x + 1) (x 2) x+1 = = . 2 x 4x + 4 (x 2) (x 2) x2 d) O numerador j a est a fatorado. Fatoremos ent ao o denominador: x2 5x + 6 = (x 2) (x 3) . (Note que x1 = 2 e x2 = 3 s ao ra zes da equa ca o quadr atica x2 5x + 6 = 0.) Ent ao, (x 3) (x 3) x3 (x 3)2 = = . 2 x 5x + 6 (x 2) (x 3) x2 Observa c ao: N ao cometa o erro: x+3 3 = . x+5 5 Esta simplica ca o est a errada, porque x n ao e um fator comum ao numerador nem ao denominador. E s o podemos simplicar fatores comuns. Atividade proposta 6.5 Simplique as fra co es abaixo: 2 2 2x + 4xy a) 2 3 , x = 0 e y = 0; x y + xy b) c) d) (a + h)3 a3 , h = 0; h (x 3) (x + 4) , x = 3; x2 6x + 9 (x + 1) (x 2)2 , x = 2 e x = 3. x2 5x + 6

56

6.4

Adi c ao de fra c oes alg ebricas

Voc e certamente se lembra de como somar fra co es num ericas de mesmo denominador: soma-se os numeradores e mant em-se o denominador. Exemplo:
2 5 3 7

+ +

8 5 5 7

= =

2+8 5 3+5 7

= =

10 5 5 7

Voc e certamente, tamb em, ainda se lembra de como somar fra co es num ericas de denominadores distintos: reescreve-se as fra co es dadas como fra co es equivalentes de mesmo denominador e efetua-se a soma. Situa c ao- problema Adicione: a) b)
1 8

1 ; 12 3 . 15

2 45

Solu c ao: a) Determinemos o m nimo m ultiplo comum (mmc) dos denominadores: 8 = 23 12 = 22 3. Lembre-se que o mmc de 8 e 12 e obtido fazendo-se o produto dos fatores comuns e n ao comuns a esses dois n umeros de maiores expoentes. Ent ao o mmc de 8 e 12 e igual a 23 .3 = 8 3 = 24. Escreve-se, ent ao, mmc (8, 12) = 24 (l e-se: o m nimo m ultiplo comum de 8 e 12 e igual a 24). Agora, observe que 1 3 1 2 = e = . 8 24 12 24 57

Desse modo, 1 3 2 5 1 + = + = . 8 12 24 24 24 b) Determinemos o mmc de 15 e 45: 15 = 3 5 45 = 32 5. Ent ao o mmc de 15 e 45 e o n umero: 32 .5 = 45. Ou seja mmc(15, 45) = 45. Notando que 3 9 = , 15 45 Obtemos: 2 3 2 9 11 + = + = . 45 15 45 45 45 Atividade proposta 6.6 Determine: 2 3 a) + 25 100 8 3 + b) 32 288 Id eias an alogas s ao utilizadas para a soma de fra co es alg ebricas: a) se as fra co es t em o mesmo denominador, ent ao soma-se os numeradores e mant em-se o denominador; b) se as fra co es tem denominadores diferentes, ent ao determina-se o mmc dos denominadores e reescreve-se as fra co es como fra co es equivalentes de mesmos denominadores e efetua-se a soma. Situa c ao-problema 6x 3 + 2x Calcule: 2x + 1 2x + 1 58

Solu c ao: Note que as fra co es t em o mesmo denominador. Ent ao 3 + 2x 6x (3 + 2x) 6x 3 2x 6x = = 2x + 1 2x + 1 2x + 1 2x + 1 4x 3 . = 2x + 1 Situa c ao-problema Adicione as seguintes fra co es alg ebricas: 3 + 2x 6x a) e ; 2x + 1 2x + 1 8 2x b) . 3 e 2 x +x (x + 1) Solu c ao: a) Observe que as fra co es alg ebricas dadas t em o mesmo dnominador, ent ao 6x 3 + 2x 3 + 8x + = . 2x + 1 2x + 1 2x + 1 b) Note que os denominadores das fra co es alg ebricas dadas s ao distintos. Determinemos, ent ao o mmc dos denominadores (x + 1)3 e x2 + x : o primeiro denominador: o segundo denominador: (x + 1)3 x2 + x = x (x + 1) .

Logo, o mmc de (x + 1)3 e x2 + x e obtido multiplicando-se os fatores comuns e n ao comuns de ambos os denominadores de maiores expoentes: x (x + 1)3 . Ent ao 8 2x 8x 2x (x + 1)2 8x + 2x (x + 1)2 + = + = (x + 1)3 x2 + x x (x + 1)3 x (x + 1)3 x (x + 1)3 Situa c ao-problema 4x 3 Adicione . 2 3 e 3 (x + 2) (x + 1) (x + 2) (x + 1)2 Solu c ao: Note que os denominadores dessas fra co es alg ebricas s ao distintos. O mmc, como se sabe, e obtido fazendo-se o produto dos fatores comuns de maiores expoentes e dos fatores n ao-comuns com tamb em os maiores expoentes: 59

primeiro denominador:

(x + 2)2 (x + 1)3

segundo denominador: (x + 2)3 (x + 1)2 . Logo, o mmc desses dois denominadores e igual a (x + 2)3 (x + 1)3 . Desse modo, 4x 3 4x (x + 2) 3 (x + 1) 2 3 + 3 2 = 3 3 + (x + 2) (x + 1) (x + 2) (x + 1) (x + 2) (x + 1) (x + 2)3 (x + 1)3 4x (x + 2) + 3 (x + 1) . = (x + 2)3 (x + 1)3 Atividade proposta 6.7 Adicione: 8 + 2a 16 + 3a + ; a) x+1 x+1 8 + 2a 16 + 3a ; b) x+1 x+1 3m + 2 m c) 3 ; x (x + 1) (x + 1)2 7 x + . d) 4 x (x + 1) x2 (x + 2)2

60

Cap tulo 7 Polin omios


Objetivo principal: conceituar polin omios; aprender a adicionar, subtrair,

multiplicar e dividir polin omios, identicando as suas principais propriedades operat orias; aprender algumas t ecnicas de determina ca o das ra zes racionais de um polin omio. Vamos iniciar esta aula denindo o que entendemos por polin omio. Esse conceito e de fundamental import ancia em toda matem atica. Ao longo do seu curso de Licenciatura em Matem atica voc e ver a que a solu ca o de muitos problemas matem aticos depende da determina ca o de ra zes de um polin omio ou da aplica ca o de suas propriedades.

7.1

Conceito de polin omio

Um polin omio em uma inc ognita (ou indeterminada ) X e a coecientes reais e uma express ao formal do tipo

P (X ) = an X n + an1 X n1 + . . . + a1 X + a0 , onde os coecientes an , an1 , . . . , a1 , a0 s ao n umeros reais e n e um n umero inteiro maior do que ou igual a zero. Se an = 0 , dizemos que n e o grau do polin omio P (X ) e escrevemos gr(P ) = n (l e-se: grau de P e igual a n) e que an e o termo l der do polin omio P (X ).

60

Quando an = 1, dizemos que o polin omio P (X ) e m onico. O n umero a0 e chamado termo independente do polin omio P (X ). Exemplos: a) P (X ) = 5X 4 + 2X 3 5X + 1 e um polin omio de grau 4 em uma inc ognita X a coecientes em R que n ao e m onico, porque a4 = 1. e um polin omio m onico de grau 2 a coecientes reais. b) D (X ) = X 2 2X + 3 Observe que um polin omio e constitu do de uma soma de mon omios. O conjunto de todos os polin omios em uma inc ognita X a coecientes reais e denotado por R[X ]. No que se segue, deniremos as opera co es de adi ca o, diferen ca, produto e quociente de polin omios em R[X ], exceto em algumas situa co es isoladas quando, ent ao, faremos uma men ca o em contr ario.

7.2
7.2.1

Opera c oes com polin omios


Adi c ao de polin omios

A soma de dois polin omios P (X ) e Q(X ) se faz adicionando-se os seus mon omios constituintes de mesmo grau. Exemplo: Considere os polin omios

P (X ) = 2 X 3 4 X 2 + 2 X + 1 Q (X ) = X 3 2 X 2 3 X + 2 Ent a o P (X ) + Q (X ) = 3 X 3 6 X 2 X + 3 . Exemplo: Se P (X ) = 2X 3 + X 3 e Q (X ) = X 2 + X + 1, ent a o P (X ) + Q (X ) = 2X 3 + X 2 + 2X 2 61

7.2.2

O produto de um n umero e de um polin omio

Se P (X ) = an X n + an1 X n1 + . . . + a1 X + a0 e um polin omio de R[X ] e k e um n umero real, dene-se o produto de k e P (X ) da seguinte maneira:

k P (X ) = kan X n + kan1 X n1 + . . . + ka1 X + ka0 Exemplo: Seja P (X ) = 2X 3 + X 3. Ent ao 5 P (X ) = 10X 3 + 5X 15.

7.2.3

A diferen ca de polin omios

Se P (X ) e Q(X ) s ao dois polin omios, dene-se a diferen ca de P e Q do seguinte modo: P (X ) Q (X ) = P (X ) + (1) Q (X ) Exemplo: Sejam P (X ) = X 5 + X 4 X + 1 e Q (X ) = X 3 2X 3. Ent ao P (X ) Q (X ) = X 5 + X 4 X + 1 + (1) X 3 2X 3 = X 5 + X 4 X + 1 X 3 + 2X + 3 = X 5 + X 4 X 3 + X + 4. Atividade proposta 7.1 Sejam P (X ) = X 3 + 4X 3 e Q (X ) = 2X 4 + X 2 1. Calcule: a) P (X ) + Q (X ) ; b) P (X ) Q (X ) ; c) 3 P (X ) 2 Q (X ) .

7.2.4

O produto de dois polin omios

O produto de dois polin omios se faz aplicando-se seguidamente a propriedade distributiva.

62

Exemplo: Se P (X ) = X 3 + X + 2 e Q (X ) = X + 3, ent ao P (X ) Q (X ) = X 3 + X + 2 (X + 3) = X3 + X + 2 X + X3 + X + 2 3 = X 4 + X 2 + 2X + 3X 3 + 3X + 6 = X 4 + 3X 3 + X 2 + 5X + 6 Ou seja X3 + X + 2 X +3 X 4 + X 2 + 2X (obtido multiplicando-se X por X 3 + X + 2) 3X 3 + 3X + 6 (obtido multiplicando-se 3 por X 3 + X + 2) X 4 + 3X 3 + X 2 + 5X + 6 (Obtido pela adi ca o de X 4 + X 2 + 2X e 3X 3 + 3X + 6) Observa c ao: Sejam P e Q dois polin omios tais que gr (P ) = m e gr (Q) = n. Ent ao o grau do produto P Q e igual a m + n, ou seja, gr (P Q) = gr (P ) + gr (Q) . No exemplo anterior, o gr (P ) = 3 e o gr (Q) = 1. Note que gr (P Q) = 3 + 1 = 4. Atividade proposta 7.2 a) Sejam P (X ) = X 3 2X 2 2 e Q (X ) = 3X 4 + X 2 . .P (X ) Q (X ) . b) Sejam P (X ) = X 3 2X 2 2 e Q (X ) = 3X 4 X 2 + 1 Calcule o produto P (X ) Q (X ) . Calcule o produto

7.2.5

O polin omio identicamente nulo

Um polin omio e identicamente nulo quando todos os seus coecientes s ao nulos. Escrevemos P (X ) 0 para denotar um polin omio P (X ) identicamente nulo. 63

Situa c ao-problema Seja P (X ) = (m2 4) X 3 +(n + 3) X +(p3 + 8). Sabendo que P (X ) e um polin omio identicamente nulo, calcule m, n e p. Solu c ao: Para que o polin omio dado seja identicamente nulo seus coecientes devem ser nulos, Ent ao devemos ter m2 4 = 0, n + 3 = 0 e p3 + 8 = 0. Donde conclu mos que m = 2, n = 3 e p = 2.

7.2.6

Quociente de polin omios

A determina ca o do quociente de dois polin omios se baseia na seguinte propriedade dos polin omios, conhecida como Algoritmo de Euclides ou Algoritmo da Divis ao : Dados dois polin omios quaisquer P (X ) e D (X ) sempre e poss vel determinar-se dois polin omios Q(X ) e R(X ) de modo que P (X ) = Q (X ) D (X ) + R (X ) ,

onde R(X ) e identicamente nulo ou o grau de R(X ) satisfaz a desigualdade 0 gr (R) < gr (D ) Chamamos P (X ) de dividendo, D (X ) de divisor, Q(X ) de quociente e R(X ) de resto da divis ao. Observa c ao: Costuma-se dispor esses polin omios da seguinte maneira: P (X ) D (X ) R (X ) Q (X )

Essa propriedade da divis ao de polin omios ser a demonstrada em um curso de Algebra Abstrata que voc es far ao posteriormente. Por enquanto nos contentaremos em aplica-la em algumas situa co es-problema. 64

Situa c ao-problema Determine o quociente e o resto do quociente dos polin omios P (X ) = 3X 5 + X 3 4 X 2 + 7 X 5 e D (X ) = X 3 4 X 2 + 7 X 5 . Solu c ao: 1a etapa: Divide-se o mon omio de maior grau do dividendo P (X ) pelo mon omio de maior grau do divisor (Neste caso, divide-se 3X 5 por X 3 obtendo-se 3X 2 ):
3 2 3 X 5 +X 4 X + 7 X 5 2 X 3 4X + 7X 5

3X 2 2a etapa: Subtrai-se do dividendo, o produto do quociente 3X 2 pelo divisor X 3 4X 2 + 7X 5 : 3X 5 + X 3 4X 2 + 7X 5 X 3 4X 2 + 7X 5 3X 2

3X 5 + 12X 4 21X 3 + 15X 2 (produto de 3X 2 pelo divisor) 12X 4 20X 3 + 11X 2 5 (primeiro resto parcial)

3a etapa: Divide-se o mon omio de maior grau do primeiro resto parcial pelo mon omio de maior grau do divisor (neste caso divide-se 12X 4 por X 3 obtendo-se 12X ) : 3X 5 + X 3 4X 2 + 7X 5 3X 5 + 12X 4 21X 3 + 15X 2
3 2 12X 4 20X + 11X 5 (primeiro resto parcial) 2 X 3 4X + 7X 5

3X 2

12X

4a etapa: Subtrai-se do primeiro resto parcial o produto de 12X pelo divisor D (X ) = X 3 4X 2 + 7X 5 : 3X 5 + X 3 4X 2 + 7X 5 3X 5 + 12X 4 21X 3 + 15X 2 12X 4 20X 3 + 11X 2 + 7X 5 (primeiro resto parcial) 12X 4 + 48X 3 84X 2 + 60X (produto de 12X pelo divisor) 28X 3 73X 2 + 67X 5 (segundo resto parcial) 5a etapa: Divide-se o mon omio de maior grau do segundo resto parcial pelo mon omio de maior grau do divisor (neste caso divide-se 28X 3 por X 3 obtendo-se 28) : 65 X 3 4X 2 + 7X 5 3X 2 + 12X

3X 5 + X 3 4X 2 + 7X 5 3X 5 + 12X 4 21X 3 + 15X 2 12X 4 20X 3 + 11X 2 + 7X 5 12X 4 + 48X 3 84X 2 + 60X
2 28X 3 73X + 67X 5(segundo resto parcial)

2 X 3 4X + 7X 5

3X 2

+ 12X

28

6a etapa: Subtrai-se do segundo resto parcial o produto de 28 pelo divisor D (X ) = X 3 4X 2 + 7X 5: 3X 5 + X 3 4X 2 + 7X 5 3X 5 + 12X 4 21X 3 + 15X 2 12X 4 20X 3 + 11X 2 + 7X 5 (primeiro resto parcial) 12X 4 + 48X 3 84X 2 + 60X 28X 3 73X 2 + 67X 5 (segundo resto parcial) 28X 3 + 112X 2 196X + 140 (produto de 28 pelo divisor) 59X 2 + 129X 135 (resto) Observe que o grau do resto R (X ) = 59X 2 + 129X 135 e igual a 2, portanto menor do que o grau do divisor, o qual e igual 3. Atividade proposta 7.3 Determine o quociente e o resto da divis ao do polin omio P (X ) = 2X 5 3X 4 + 9X 2 + 12X 135. pelo polin omio D (X ) = X 2 + 2X . X 3 4X 2 + 7X 5 (divisor) 3X 2 + 12X + 28 (quociente)

7.3

Raiz ou zero de um polin omio

Seja P (X ) = an X n + an1 X n1 + . . . + a1 X + a0 um polin omio de grau n. O valor do polin omio P no n umero real r e o n umero real P (r ) obtido ao substituirmos X por r na express ao de P (r ), ou seja, calculando-se P (r ) = an r n + an1 r n1 + . . . + a1 r + a0. 66

Exemplo: Os valores do polin omio P (X ) = X 3 + 2X + 1 nos n umeros r = 1, r = 0 e r = 1 s ao dados por: P (1) = (1)3 + 2 (1) + 1 = 2; P (0) = 03 + 2 0 + 1 = 1 = 1; P (1) = 13 + 2 1 + 1 = 4. Dizemos que um n umero real r e um zero ou uma raiz de um polin omio P (X ) quando P (r ) = 0, ou seja, quando o valor de P em r e igual a zero. Situa c ao-problema Dado o polin omio P (X ) = X 3 5X 2 + 10X 6 , pergunta-se: a) x0 = 2 e um zero (uma raiz) de P (X )? b) x0 = 1 e um zero (uma raiz) de P (X )? Solu c ao: a) Calculemos P(2): P (2) = 23 5 22 + 10 2 6 = 2. Como P (2) = 0, conclu mos que 2 n ao e um zero (uma raiz) de P (X ). b) Calculemos P (1): P (1) = 13 5 12 + 10 1 6 = 0. Como P (1) = 0, conclu mos que X = 1 e um zero (uma raiz) de P (X ).

7.4

Divisibilidade de polin omios

Dizemos que um polin omio P (X ) e divis vel por um polin omio D (X ) quando o resto da divis ao de P (X ) por Q(X ) e igual a zero. Desse modo, P (X ) e divis vel pelo polin omio D (X ) quando P (X ) = Q (X ) D (X ) para algum polin omio Q(X ). Neste caso dizemos que P (X ) = Q (X ) , D (X ) ou seja, que P (X ) dividido por D (X ) e igual a Q(X ). 67

Situa c ao-problema Mostre que o polin omio X 4 a4 e divis vel pelo polin omio X a e que X 4 a4 = X 3 + aX 2 + a2 X = a3 . X a

Solu c ao: X 4 a4 X 4 + aX 3 aX 3 a4 aX 3 + a2 X 2 a2 X 2 a4 a2 X 2 + a3 X a3 X a4 a3 X + a4 0 Atividade proposta 7.4 a) Mostre que o polin omio X 5 a5 e divis vel pelo polin omio X a e que X 5 a5 = X 4 + aX 3 + a2 X 2 + a3 X + a4 . X a b) Mostre que o polin omio X n an , n 1 e divis vel pelo polin omio X a e que X n an = X n1 + X n2a + X n3 a2 + . . . + Xan2 + an1 . Xa X a X 3 + aX 2 + a2 X + a3

Situa c ao-problema Mostre que se um polin omio P (X ) e divis vel por X r , ent a o P (r ) = 0 . 68

Solu c ao: Se P (X ) e divis vel por X r , ent ao P (X ) = Q (X ) (X r ) Donde conclu mos que P (r ) = Q (r ) (r r ) = Q (r ) 0 = 0. Situa c ao-problema: Mostre que se para algum polin omio P (X ) tivermos P (r ) = 0 para algum n umero real r , ent a o P (X ) e divis vel por X r . Solu c ao: Pelo Algoritmo da Divis ao, sabemos que P (X ) = Q (X ) (X r ) + R (X ) , onde R(X ) e um polin omio constante, porque o divisor D (X ) = X r tem grau 1 e o grau do resto e menor do que o grau do divisor ou R (X ) 0. Por hip otese, sabemos que P (r ) = 0, ent ao P (r ) = Q (r ) (r r ) + R (r ) . (7.1)

Como P (r ) = 0, conclu mos da igualdade ( 7.1) acima que R(r ) = 0. Como R(X ) e constante, e R(X ) se anula em X = r, ent ao R(X ) = 0 para todo X . Da segue-se que P (X ) = Q (X ) (X r ) , mostrando assim que P (X ) e divis vel por X r. Situa c ao-problema Mostre que o polin omio P (X ) = X 100 5X 32 + 10X 6 e divis vel pelo polin omio X 1. Solu c ao: Basta observar que P (1) = 1100 5 132 + 10 1 6 = 0. Pela situa c ao-problema anterior, conclu mos que P (X ) e divis vel por X 1. 69

7.5

O resto da divis ao de um polin omio por X r

Situa c ao-problema Seja r um n umero real qualquer. Mostre que o resto da divis ao de P (X ) por X r e igual a P (r ). Solu c ao: Seja D (X ) = X r . Ent ao pelo Algoritmo da Divis ao temos que P (X ) = Q (X ) D (X ) + R (X ) (7.2)

onde R(X ) e identicamente nulo ou o grau de R(X ) satisfaz a desigualdade 0 gr (R) < gr (D ) Ent ao, como o grau do polin omio D (X ) e igual a 1, conclu mos que R (X ) 0 ou o grau de R(X ) e igual a zero. Se o grau de R(X ) for igual a zero, ent a o R (X ) e um polin omio constante, isto e, R(X ) = k , onde k e uma constante. Na hip otese de R (X ) 0, obtemos da igualdade ( 7.2) acima, a seguinte igualdade: P (X ) = Q (X ) (X r ) . Da segue-se que P (r ) = Q (r ) (r r ) = Q (r ) 0 = 0. Como P (r ) = 0 e R(X ) =0, conclu mos que R(X ) = P (r ). Ou seja, que o resto da divis ao de P (X ) por X r e igual a P (r ). Na hip otese de o grau de R(X ) ser igual a k , segue-se da igualdade ( 7.2) acima a seguinte identidade:

P (X ) = Q (X ) (X r ) + k Desse modo, P (r ) = Q (r ) (r r ) + k = Q (r ) 0 + k. Donde segue-se que P (r ) = k . Como R(X ) = k , conclu mos que R(X ) = P (r ). Mostrando assim que o resto R(X ) da divis ao de P (X ) por X r e igual a P (r ). 70

Atividade proposta 7.5 a) Seja P (X ) = 3X 4 5X 3 + 10X 6. Aplicando a situa ca o-problema anterior, determine o resto da divis ao de P (X ) pelo polin omio D (X ) = X + 2. b) Dividindo-se P (X ) = X 2 + kX + 2 por X + 1 e X 1 s ao encontrados restos iguais. Ache o valor de k.

7.6

Determina c ao polin omio

das

ra zes

racionais

de

um

Nesta se ca o, trataremos da quest ao da determina ca o das ra zes racionais de um polin omio. Como ser a visto logo adiante, essa propriedade ser a muito u til na fatora ca o dos polin omios que possuem tais ra zes. Iniciaremos enunciando um fato acerca da divisibilidade de um produto de n umeros inteiros por um n umero primo: Se um n umero primo p divide um produto de n umeros inteiros ab, ent ao p divide a ou p divide b.

(A prova desse resultado ser a feita em um curso de Teoria dos N umeros posteriormente). Situa c ao-problema Seja P (X ) = an X n + an1 X n1 + . . . + a1 X + a0 um polin omio em uma indeterminada X e a coecientes inteiros Mostre que se P (X ) tem uma raiz racional do tipo r = ent ao p e um divisor de e q e um divisor de. (Lembre-se: dois n umeros naturais p e q s ao primos entre si quando eles n ao t em fator comum maior do que 1, ou seja, quando o mdc de p e q e igual a 1). Solu c ao: 71
p q

com p e q primos entre si,

Suponha que r =

p q

e uma raiz de P (X ), com p e m primos entre si. Como m e raiz de


p q

P (X ), devemos ter P (r ) = 0, ou seja, P Da segue-se que p q p q


n

= 0.

= an

+ an1

p q

n 1

+ . . . + a1

p q

+ a0 = 0.

Logo, sabemos que pn pn 1 p + a + . . . + a1 + a0 = 0. n 1 n n 1 q q q

an

Multiplicando ambos os membros desta u ltima equa c ao por q n , obtemos

an pn + an1 pn1 q + . . . + a1 pq n1 + a0 q n = 0. Donde,

(7.3)

an pn + an1 pn1 q + . . . + a1 pq n1 = a0 q n . Colocando o fator comum p em evid encia no primeiro membro dessa u ltima equa c ao, obtemos a seguinte igualdade: p an pn1 + an1 pn2 q + . . . + a1 q n1 = a0 q n . Como p divide o primeiro membro desta u ltima equa c ao, ent ao p tamb em divide o seu segundo membro. Mas p n ao divide q n (porque se p dividisse uma pot encia de q , ent ao p dividiria q , o que contrariaria a hip otese de p e q serem primos entre si). Assim, p divide a0 . Para mostrar, agora, que o n umero q deve dividir an , reescrevamos a equa c ao (7.3) acima do seguinte modo:

an1 pn1 q + . . . + a1 pq n1 + a0 q n = an pn .

72

Coloquemos q em evid encia no primeiro membro da equa c ao pelo m etodo de fatora c ao do fator comum:

q an1 pn1 + an2 pn2 q + . . . + a1 pq n2 + a0 q n1 = an pn . Observe, agora, que q divide o primeiro membro desta u ltima equa c ao. Ent ao, q tamb em divide o seu segundo membro. Mas como q n ao divide pn , conclu mos que q divide o fator an . Isso conclui a solu c ao da situa c ao-problema proposta. Situa c ao-problema Determine as ra zes racionais do polin omio P (X ) = 2X 3 6X 2 X + 3 caso elas existam. Solu c ao: Pelo que vimos na situa c ao-problema anterior, se existirem ra zes racionais de P (X ) elas ser ao do tipo r =
p q

com p e q primos entre si e q = 0, onde p e um divisor de 3 e q

e um divisor de 2. Ent ao p {1, 3} e q {1, 2} Logo, as poss veis ra zes de P (X ) , 3 pertencem ao conjunto 1, 1 2 P (X ) e X = 3. Observa c ao: Vale observar que, como P (3) = 0, ent a o P (X ) e divis vel por X 3. Isso permite a fatora ca o de P (X ) na forma. P (X ) = Q (X ) (X 3) Para determinarmos Q(X ), apliquemos o Algoritmo da Divis ao: 2X 3 6X 2 X + 3 2X 3 + 6X 2 X + 3 X 3 0 73 X 3 2X 2 1
3 2

f E acil vericar que P (X ) s o se anula para X = 3. Portanto, a u nica raiz racional de

Portanto, Q (x) = 2X 2 1 . Assim P (X ) pode ser fatorado na forma P (X ) = 2X 2 1 (X 3) . Atividade proposta 7.6 Determine, se existirem, as ra zes racionais de cada uma dos seguintes polin omios: a) X 4 + X 3 + X 2 X 2; b) 5X 5 + X 2 X 2. Situa c ao-problema Seja .X 4 5X 3 + X 2 + 5X 2; a) Determine todas as ra zes racionais de P (X ) ; b) fatore P (X ) em Z[X ], isto e, no conjunto dos polin omios em uma indeterminada X e a coecientes inteiros; c) fatore P (X ) em R[X ], isto e, no conjunto dos polin omios em uma indeterminada X e a coecientes reais. Solu c ao: a) Observe inicialmente que os coecientes de P (X ) s ao inteiros. Desse modo, se r =
p q

e uma raiz racional de P (X ), com p e q inteiros primos entre si, com q n ao nulo, ent ao p {1, 1, 2, 2} e p {1, 1}. Logo,
p q

f {1, 2}. E acil vericar que

P (1) = 0, P (1) = 0, P (2) = 16 e P (2) = 4. Assim as u nicas ra zes racionais de P (X ) s ao X = 1 e X = 1. b) Pelo item anterior, sabemos que P (X ) e divis vel por X 1 e X + 1. Apliquemos ent ao o Algoritmo da Divis ao para determinarmos o quociente Q1 (X ) de P (X ) por X 1:

74

X 4 5X 3 + X 2 + 5X 2 X 4 + X 3 4X 3 + X 2 + 5X 2 4X 3 4X 2 3X 2 + 5X 2 3X 2 3X 2X 2 2X + 2

X 1 X 3 4X 2 3X + 2

0 Logo, Q1 (X ) = X 3 4X 2 3X + 2. Desse modo, obtemos a seguinte igualdade: P (x) = Q1 (X ) (X 1) = X 3 4X 2 3X + 2 (X 1) . Note agora que Q1 (1) = (1)3 4 (1)2 3 (1) + 2 = 0. Ent ao Q1 (X ) e divis vel por X + 1. Aplicando novamente o Algoritmo da divis ao obtemos: X 3 4X 2 3X + 2 X 3 X 2 5X 2 3X + 2 5X 2 + 5X 2X + 2 2X 2 0 Da segue-se que X +1 X 2 5X + 2

X 3 4X 2 3X + 2 = X 2 5X + 2 (X + 1) . Portanto, temos a seguinte fatora c ao de P (X ) em Z[X ]: X 4 5X 3 + X 2 + 5X 2 = X 2 5X + 2 (X + 1) (X 1) . c) Observe que X 2 5X + 2 possui as seguintes ra zes: 75

5 b X= = 2a

(5)2 4 1 2 21

17

Ent ao em R[X ], o polin omio X 4 5X 3 + X 2 + 5X 2 se fatora da seguinte maneira: X 5X + X + 5X 2 =


4 3 2

5+

17

17

(X + 1) (X 1)

7.7

Rela c oes entre coecientes e ra zes

Quando estudamos as equa co es quadr aticas, vimos que se r1 e r2 s ao ra zes de uma equa ca o quadr atica ax2 + bx + c = 0, a = 0, ent ao

ax2 + bx + c = a (x r1 ) (x r2 ) . A partir dessa u ltima igualdade e f acil vericar que r1 + r2 = b a c r1 r2 = . a

Essas duas u ltimas igualdades s ao conhecidas como rela co es de Girard para uma equa ca o quadr atica. Elas expressam as rela co es entre as ra zes de uma equa ca o quadr atica e os seus coecientes. Situa c ao-problema Sejam r1 e r2 as ra zes da equa ca o 2x2 + a) r1 + r2 ; b) r1 r2 . Solu c ao: 3x + 2 = 0. Calcule:

76

Na situa c ao-problema seguinte, veremos que existem rela c oes an alogas entre os coecientes e as ra zes de um polin omio de grau 3. Situa c ao-problema Seja P (X ) = aX 3 + bX 2 + cX + d um polin omio de grau 3 (a = 0) Suponha que P (X ) pode ser fatorado do seguinte modo: aX 3 + bX 2 + cX + d = a (X r1 ) (X r2 ) (X r3 ) .

Pelas rela c oes de Girard, obtemos 3 r1 + r2 = 2 r r = 2. 1 2 2

Mostre que r1 , r2 e r3 s ao ra zes de P (X ) e que b r1 + r2 + r3 = a c r1 r2 + r1 r3 + r2 r3 = a d r1 r2 r3 = . a

Solu c ao: Para ver que r1 e raiz de P (X ), basta observar que

P ( r1 ) = a ( r1 r1 ) ( r1 r2 ) ( r1 r2 ) = 0 . Analogamente, se mostra que

P ( r1 ) = P ( r3 ) = 0 . Donde segue-se que r1 , r2 e r3 s ao ra zes de P (X ).

77

Note, agora, que o primeiro membro da equa c ao dada no enunciado da situa c ao-problema pode ser reescrito na forma c d b aX 3 + bX 2 + cX + d = a X 3 + X 2 + X + a a a

Por outro lado, o segundo membro da equa c ao dada no enunciado da situa c ao-problema pode ser reescrito como a ( X r1 ) ( X r2 ) ( X r3 ) = a X 3 ( r1 + r2 + r3 ) X 2 + ( r1 r2 + r1 r3 + r2 r3 ) X + ( r1 r2 r3 ) . Igualando-se os dois membros da equa c ao do enunciado, obtemos o resultado que queremos: b r1 + r2 + r3 = a c r1 r2 + r1 r3 + r2 r3 = a r1 r2 r3 = d . a 2X 2 3X + 3 2, sabendo que

Situa c ao-problema

Determine as ra zes do polin omio P (X ) = X 3 r e r , onde r e um n umero real)

ele tem duas ra zes sim etricas (isto e, sabendo que duas ra zes de P (X ) s ao do tipo .

Solu c ao: Sejam r , r e s as ra zes de P (X ). Ent ao, pelas rela c oes de Girard, temos que r + (r ) + s = 2 Da primeira equa c ao, obtemos s = Basta vericar agora que 2X 2 3X + 3 2.

r (r ) + r s + s r = 3 r (r ) s = 3 2. 2. Da segunda equa c ao, obtemos s = 3.

X+

3 = X3

78

O resultado desejado decorre, agora, da situa c ao-problema anterior. Observa c ao: Para um polin omio de grau n tamb em existem rela co es entre as ra zes e os coecientes de um polin omio, as quais s ao dadas a seguir. Seja P (X ) = an X n + an1 X n1 + . . . + a1 X + a0 um polin omio de grau n (an = 0) . Suponha que P (X ) pode ser fatorado do seguinte modo:

an X n + an1 X n1 + . . . + a1 X + a0 = an (X r1 ) (X r2 ) . . . (X rn ) . f E acil vericar que s ao ra zes de P (X ) e que an1 r1 + r2 + . . . + rn = an an2 r1 r2 + r1 r3 + . . . + rn1 rn = (1)2 an

Essas rela co es s ao chamadas rela co es de Girard para um polin omio de grau n. Situa c ao-problema Aplicando as rela co es de Girard, encontre um polin omio de grau 3 cujas ra zes s ao 1, 2 e 3. Solu c ao: Seja P (X ) = aX 3 + bX 2 + cX + d o polin omio de grau 3 que queremos determinar. As tr es ra zes do polin omio devem satisfazer as seguintes rela c oes : b 1 + 2 + 3 = b = 6a a c 1 2 + 1 3 + 2 3 = c = 11a a d 1 2 3 = d = 6a. a Logo, qualquer polin omio do tipo 79

an3 r1 r2 r3 + r1 r2 r4 + r1 r2 r5 + . . . + rn2 rn1 rn = (1)3 an . . . a0 r1 r2 r3 . . . rn = (1)n . an

P (X ) = aX 3 6aX 2 + 11aX + 6a, onde a e um n umero real qualquer, tem como ra zes 1, 2 e 3. Podemos ent ao tomar, em particular, o polin omio

P (X ) = X 3 6X 2 + 11X + 6. Atividade proposta 7.7 Aplicando as rela co es de Girard, determine um polin omio de grau 4 com coeciente l der 2 cujas ra zes sejam 1, 1, 2 e 3.

80

Aula 8 Conjuntos Num ericos


Nosso objetivo central nesta aula e apresentar os conjuntos de n umeros que aparecer ao com frequ encia em aulas posteriores e que se notabilizaram pela sua relev ancia social ao longo da hist oria cultural da humanidade. Iniciaremos introduzindo, na primeira se ca o, a no ca o de conjuntos. Nas se co es seguintes introduziremos os conceitos de conjuntos naturais, inteiros, racionais, irracionais e reais.

8.1

Conjuntos: algumas no c oes preliminares

Comecemos pela no ca o de conjunto. Um conjunto e uma no ca o primitiva, isto e, uma palavra cujo signicado e admitido intuitivamente, sem a necessidade de traduzirmos o seu signicado por interm edio de outras palavras. Em nosso cotidiano, a palavra cole ca o ea que mais se aproxima do signicado matem atico da palavra conjunto. Por exemplo, o conjunto de torcedores do CSA e a cole ca o de todas as pessoas que torcem pelo CSA; o conjunto dos planetas do Sistema Solar e a cole ca o constitu da pelos seguintes corpos celestes que orbitam em torno do Sol: Merc urio, Marte, V enus, J upiter, Saturno, Urano, Netuno, Terra, Plut ao. H a basicamente dois modos de apresentarmos um conjunto: (1) listando-se todos os seus elementos; (2) enunciando um crit erio atrav es do qual se possa decidir se um elemento

80

pertence ou n ao pertence ao conjunto, como no exemplo planetas citado no par agrafo anterior. Classicamente, utilizam-se as seguintes nota co es para designar um conjunto: (1) o conjunto T dos torcedores do CSA, e representado simbolicamente da seguinte maneira: T = {x | x e torcedor do CSA} (l e-se: o conjunto T e o conjunto dos elementos x tais que x e torcedor do CSA). (2) O conjunto P dos planetas do Sistema Solar pode ser indicado como P = {x | x e um planeta do Sistema Solar} , como tamb em pode ser assim representado P = {Merc urio, Marte, Venus, J upiter, Saturno, Urano, Netuno, Terra, Plut ao} . Observa c ao: Embora, atualmente, o conceito de n umero possa ser obtido a partir de conceitos da teoria dos conjuntos, admitiremos a no ca o de n umero tamb em como primitiva.

Atividade: Quais s ao os elementos dos conjuntos S = {x / x e dia de uma semana}, T = {x / x e um sat elite da Terra} e V = {x / x e um ser humano imortal}

8.2

O conjunto dos n umeros naturais

Passemos, agora, ao conceito do conjunto dos n umeros naturais. Nosso objetivo nessa se ca o e descrever o conjunto dos n umeros naturais, xar a nota ca o que utilizaremos no restante desse curso e mencionar as suas principais propriedades operat orias. Nestas notas de aula, o conjunto dos n umeros naturais e denotado pelo s mbolo N e e constitu do pelos n umeros que no sistema de numera ca o indo-ar abico s ao representados por

0, 1, 2, 3, . . . , n, . . . 81

Na linguagem de conjuntos, N = {0, 1, 2, 3, . . .}. Dado qualquer n umero n no conjunto N = {1, 2, 3, . . .}, o n umero n 1 e chamado antecessor de n e o n umero n + 1 e chamado sucessor de n. Desse modo, por exemplo, 5 e sucessor de 4 e 3 e antecessor de 4. Situa c ao-problema: Determine os seguintes conjuntos: a) A = {x N / x > 7} e A = {x N / x 5}; b) A B = {x / x A ou x B }; c) A B = {x / x A e x B } . Atividade proposta: Determine os seguintes conjuntos: a) A = {x N / 3 < x < 10} e B = {x N / 2 x < 7} b) A B = {x / x A ou x B } c) A B = {x / x A e x B } Observa c ao: Historicamente, os n umeros naturais foram constru dos associados ao processo de contagem de elementos de cole co es de objetos. Atualmente, o conjunto dos n umeros naturais constitui-se em um eciente modelo matem atico de contagem largamente utilizado em nosso dia-a-dia. Observa c ao: O conjunto dos n umeros naturais pode ser apresentado axiomaticamente, atrav es dos axiomas de Peano; o leitor interessado nessa abordagem pode consultar (...). Observa c ao: Para o epistem ologo su co Jean Piaget, os n umeros naturais s ao

constru dos como uma s ntese de opera co es de seria ca o, classica ca o, inclus ao hier arquica por interm edio de abstra co es reexivas; o leitor interessado neste ponto de vista pode consultar (...)

8.2.1

Propriedades operat orias dos n umeros naturais

A cada par (a, b) de n umeros naturais podemos associar dois outros n umeros naturais: a + b (a adi ca o de a e b) e a b (o produto de a e b) que t em as seguintes propriedades: a + b = b + a (comutatividade) 82

(a + b) + c = a + (b + c) (associatividade) a + 0 = a (exist encia do elemento neutro da adi ca o) a b = b a (comutatividade) a (b c) = (a b) c (associatividade) a 1 = a (exist encia do elementro neutro do produto) a (b + c) = a b + a c (distributividade) Atividade: Podemos calcular o produto 4 64 aplicando a propriedade distributiva dos n umeros naturais, da seguinte maneira: 4 64 = 4 (60 + 4) = 4 60 + 4 4 = 240 + 16 = 256. Aplique a propriedade distributiva para efetuar os seguintes produtos: 5 61 e .8 53. Observa c ao: Se a, b N , ent ao a + b N e a b N ; diz-se neste caso que o conjunto dos n umeros naturais e fechado com rela ca o a ` adi ca o e a ` multiplica ca o. Contudo, o conjunto dos n umeros naturais N n ao e fechado nem com rela ca o a ` subtra ca o nem com rela ca o divis ao. Por exemplo, se a = 5 e b = 9, ent a o a b N e a b N.

8.3

O conjunto dos n umeros inteiros

Passemos, agora, ao conceito do conjunto dos n umeros inteiros. O conjunto dos n umeros inteiros ser a representado pelo s mbolo Z e e constitu do de todos os n umeros que no sistema indo-ar abico s ao representados por 0, 1, 2, 3 . . . Ou seja, na linguagem de conjuntos Z = {3, 2, 1, 0, 1, 2, 3, . . .} Observa c ao: N Z (l e-se: o conjunto dos n umeros naturais est a contido no conjunto dos n umeros inteiros). Isto signica que todo elemento de N e tamb em um elemento do conjunto Z. 83

Observa c ao: Historicamente, os n umeros negativos apareceram em registros cont abeis como s mbolos indicativos de d vidas ou faltas. Sua consagra ca o na Matem atica se deve ao fato de permitirem a amplia ca o do conjunto dos n umeros naturais de modo a tornar a opera ca o de subtra ca o sempre poss vel (cf. Boyer,...).

8.3.1

O valor absoluto de um n umero


Falemos

Trataremos agora do conceito do valor absoluto de um n umero inteiro.

inicialmente da representa ca o dos n umeros inteiros em uma reta. Como se sabe, cada n umero inteiro pode ser representado sobre uma reta na qual escolhemos um ponto O , chamado origem e ao qual associamos o n umero zero, e um outro ponto U , ao qual associamos o n umero 1, conforme mostra a gura: O 3 2 1 0 U 1 2 3

Desse modo, a cada n umero inteiro corresponder a um ponto da reta. Ao n umero 2, por exemplo, corresponder a o ponto obtido partindo-se da origem e andando-se duas unidades para a direita, isto e, no sentido de percurso escolhido como crescente. Ao n umero 3 corresponder a o ponto que se obt em partindo-se da origem e andando-se tr es unidades para a esquerda, ou seja, no sentido oposto ao sentido de percurso escolhido como crescente. Desse modo, cada n umero inteiro x determina um ponto X sobre a reta orientada. O 0 O x U 1 U X x

0 1 Denimos o valor absoluto |x| de um n umero inteiro x como sendo o comprimento do segmento OX quando adotamos como unidade de medida o comprimento do segmento OU . Desse modo, |+3| = 3 e |3| = 3; |+4| = 4 ;|4| = 4;|5| = 5.

84

8.3.2

Propriedades operat orias dos n umeros inteiros

Admitiremos as seguintes regras de opera ca o com os inteiros: 1) Adi ca o de dois n umeros positivos e um n umero positivo; exemplo: 2 + 3 = 5. Essa adi ca o corresponde, geometricamente, a sair da origem, andar duas unidades no sentido positivo at e o ponto de coordenada 2, e desse ponto andar mais tr es unidades para a direita, chegando-se assim ao ponto de coordenada 5. 2) Adi ca o de dois n umeros negativos e um n umero negativo; exemplo: 2 + (3) = 5. Essa adi ca o corresponde, geometricamente, a sair da origem, andar duas unidades no sentido negativo at e o ponto de coordenada 2, e da andar mais tr es unidades para a direita, chegando-se assim ao ponto de coordenada -5. 3) Adi ca o de n umeros de sinais contr arios: considere como motiva ca o a opera ca o 6 9 = 6 + (9) = 3 . Essa opera ca o corresponde geometricamente a andar 6 unidades no sentido positivo at e o ponto de coordenada 6, e da desse ponto andar nove unidades para a esquerda, chegando-se assim ao ponto de coordenada 3. Desse modo, para obter a adi ca o de n umeros de sinais contr arios subtrai-se do maior valor absoluto dos dois n umeros inteiros dados o menor valor absoluto dos dois n umeros dados afetando a diferen ca com o sinal do n umero de maior valor absoluto; ou seja, Exemplos: (i) 3 2 = 1; (ii) 5 + 3 = (5 3) = 2 (o valor absoluto de 5 e maior do que o de 3); (iii) 5 9 = (9 5) 4 (o valor absoluto de 9 e maior do que o de 5) 1) O produto de dois n umeros positivos e um n umero positivo: 2 4 = 8 2) O produto de dois n umeros negativos e um n umero positivo:2 (3) = 6 3) O produto de dois n umeros de sinais contr arios e um n umero negativo:

(3) 4 = 12 5 (3) = 15 Observa c ao: Essas regras s ao escolhidas assim para que as propriedades de 85

comutatividade, associatividade e de distributividade dos n umeros naturais possam ser estendidas para o conjunto dos n umeros inteiros. Observa c ao: O conjunto Z dos n umeros inteiros e fechado com rela ca o a ` adi ca o, a ` multiplica ca o e a ` subtra ca o, mas n ao e fechada com rela ca o a ` divis ao: a = 5 e b = 7, ent ao a b Z. .

8.4

O conjunto dos n umeros racionais

O conjunto dos n umeros racionais ser a denotado por Q e e constitu do de todas as fra co es , onde p e q s ao n umeros inteiros sendo q = 0 . Ou seja, Q= Observa c ao:N Z Q. Observa c ao: Historicamente, o conceito de n umeros racionais encontra-se associado ao conceito geom etrico de segmentos comensur aveis. Dois segmentos de reta AB e CD s ao comensur aveis quando existe um segmento EF que cabe um n umero inteiro de vezes em AB e um n umero inteiro de vezes em CD . Ent ao, quando AB e CD s ao comensur aveis, podemos escrever: AB = pEF (quando EF cabe p vezes em AB ) CD = qEF (quando EF cabe q vezes em CD ) Dessa maneira, o segmento AB est a para o segmento CD na mesma propor ca o em que o n umero inteiro p est a para o n umero inteiro q . A C E F 86 B D p | p Z, q Z, q = 0 q

Exemplo: Na gura acima AB = 3EF e CD = 5EF . Logo

AB CD

3 . =5

8.4.1

Opera co es com os n umeros racionais

Nesta se ca o, mencionaremos as propriedades operat orias dos n umeros racionais, as quais ocupam um lugar muito importante neste curso e na matem atica. Sejam a, b, c e d n umeros inteiros. As opera co es com os n umeros racionais s ao denidas do modo seguinte: Adi c ao: a + b
a b c b

a+c ,b b

= 0 (adi ca o de fra co es de mesmo denominador)

c d

ab+cb ,b bd a b

= 0, d = 0 (adi ca o de fra co es de denominadores quaisquer)


c b

Subtra c ao:
a b

c d

ac ,b b

= 0 (subtra ca o de fra co es de mesmo denominador)

c d

adcb ,b bd a b

= 0 (subtra ca o de fra co es de denominadores quaisquer) = =


ac ,b bd a b

Produto:

= 0, d = 0.

Quociente:

a b c d

d , b = 0, c = 0, d = 0. c

Situa c ao-problema: Localize na reta num erica orientada os seguintes n umeros


1 3 7 5 1 racionais: 2 , 4, 5, 3 , 3

Solu c ao: Cada n umero racional corresponde a um ponto de uma reta: 5 3 1 3 0


13 24 7 5

Situa c ao problema: Determine o valor da express ao num erica Solu c ao: Basta observar, inicialmente, que 2 4 23+45 6 + 20 26 + = = = 5 2 53 15 15 Ent ao, 2 4 1 26 1 26 2 1 15 37 + = = = 5 2 2 15 2 15 2 30 87

2 5

+4 1 . 3 2

Atividade: Determine o valor da express ao num erica

2 5

4 + 3

1 2

Situa c ao-problema: mostre que a soma de dois n umeros racionais e um n umero racional Solu c ao: Sejam
a b

c d

, dois n umeros racionais quaisquer. Ent ao, a c ab + cb + = b d bd

Como o conjunto dos inteiros e fechado com rela ca o ao produto e a ` adi ca o, conclu mos que ab + cb Z e que bd Z; como b = 0 e d = 0 , conclu mos tamb em que bd = 0. Portanto,
ab+cb bd

e um n umero racional, mostrando assim o resultado que quer amos.

Atividade: a) Mostre que o produto de dois n umeros racionais e um n umero racional; b) Mostre que o quociente de dois n umeros racionais e um n umero racional. Situa c ao-problema: Encontre um n umero racional entre os n umeros Solu c ao:
1 3

e 1 . 2

Suponha que A e B s ao os pontos sobre a reta que correspondem,


1 3

respectivamente, aos n umeros racionais

1 2

. Ent ao o ponto m edio M , isto, e, situado

exatamente na metade do segmento AB , corresponde ao n umero racional


1 3

+ 2

1 2

12+13 23

5 6

5 1 5 = . 6 2 12

Atividade: Encontre um n umero racional entre os n umeros: (a) 2 e 3 ; 5 5


2 3 (b) 5 e 5 .

Atividade: Mostre que entre dois n umeros racionais sempre existe um outro racional.

88

8.4.2

Decimais peri odicos

N umeros decimais, tais como 0, 444. . . e 0, 252525 . . ., que ap os a v rgula apresentam algarismos que se repetem periodicamente s ao chamados d zimas peri odicas. Veremos nesta se ca o que tais d zimas s ao n umeros racionais. Situa c ao-problema: Mostre que: (a) 0, 444 . . . = 4 ; 9 (b) 0, 252525 . . . = (c) 3, 888 . . . = Solu c ao: (a) Fa ca x = 0, 444 . . .Ent ao 10x = 4, 444 . . . = 4 + 0, 444 . . . = 4 + x. Da obtemos 9x = 4, donde se segue que x = 4 , que e o resultado que quer amos. 9 (b) Fa ca x = 0, 252525 . . . Ent ao 100x = 25, 252525 . . . = 25 + 0, 252525 . . . = 25 + x. Da obtemos 99x = 25, donde se segue que x =
25 , 99 35 . 9 25 ; 99

que e o resultado que quer amos.

(c) Observe inicialmente que 3, 888 . . . = 3 + 0, 888 . . . Fa ca x = 0, 888 . . . Ent ao 10x = 8, 888 = 8 + x.
8 Logo 9x = 8. Donde se segue que 0, 888 . . . = 9 . Portanto

3, 888 . . . = 3 + 0, 888 . . . = 3 + que e o resultado que quer amos. Atividade: (a) Mostre que 0, 999 . . . = 1; (b) Mostre que 2, 999 . . . = 3.

8 35 = , 9 9

89

8.5

O conjunto dos n umeros irracionais

Passemos agora ao conceito de n umeros irracionais. Para falar sobre esses n umeros, vamos precisar da no ca o de segmentos incomensur aveis. Dizemos que dois segmentos AB e CD s ao incomensur aveis se n ao existe nenhum segmento EF que cabe um n umero inteiro de vezes em AB e um n umero inteiro de vezes em CD . Um n umero irracional e um n umero que representa o comprimento de um segmento de reta que e incomensur avel com rela ca o a qualquer outro segmento de reta. Situa c ao-problema: (a) Mostre que a diagonal de um quadrado de lado unit ario mede (b) Mostre que 2 e um n umero irracional. Solu c ao: (a) O comprimento da diagonal do quadrado pode ser obtido por interm edio do teorema de Pit agoras: se os lados do tri angulo t em comprimento 1, ent ao o comprimento d da diagonal do quadrado e dado por d = 12 + 12 , Donde conclu mos que d = (b) Suponha que 2. 2.

2 e um n umero racional p , q = 0, com p e q n umeros primos entre q si (isto e, 1 e o m aximo divisor comum de p e q ). Ou seja, estamos supondo que 2 = p . q 2 2 Da segue-se que 2 = p . Ent ao, p2 = 2q 2 . Conclu mos da que p2 e um n umero par, q2 donde obtemos tamb em que q e um n umero par (veja a observa ca o abaixo). Desse modo, p = 2k , para algum n umero natural k . Da igualdade p2 = 2q 2 , segue-se que (2k )2 = 2q 2 . Logo, q 2 = 2k 2 . Donde segue-se que o n umero q e tamb em um n umero par. Desse modo, 2 seria um divisor comum de p e q , contrariando o fato de p e q serem primos entre si. Essa contradi ca o proveio da hip otese de 2 ser um n umero racional. Como 2 n ao pode ser racional, s o resta uma alternativa: e irracional. Observa c ao: Um n umero natural n e par quando pode ser escrito na forma n = 2k , 90

com k N , e mpar quando pode ser escrito na forma n = 2k + 1 , com k N. Mostremos que se o quadrado de um n umero natural e par, ent ao ele n ao pode ser mpar. De fato, suponha que p2 e par e que p = 2m + 1, com m N . Ent ao, p2 = 4m2 + 4m + 1 = 2 2m2 + 2m + 1 = 2k + 1, onde k = 2 2m2 + 2m N, donde conclu mos que se p e mpar, ent ao p2 tamb em e mpar. Obter amos, assim, uma contradi ca o: p2 seria par e mpar. Portanto, se p2 for par, ent ao p n ao pode ser mpar.

8.6

O conjunto dos n umeros reais

Passemos agora ao conceito de n umeros reais. O conjunto dos n umeros reais e denotado pelo s mbolo R e e constitu do pela reuni ao dos n umeros racionais e dos n umeros irracionais. Ou seja,

R = {x | x e um n umero racional ou x e um n umero irracional } Observa c ao: N Z Q R.

8.6.1

A reta real

Costuma-se representar geometricamente o conjunto dos n umeros reais por interm edio de uma reta, na qual se escolhem um ponto O denominado origem (ao qual se associa o n umero zero 0) e um ponto U (ao qual se associa o n umero 1). O ponto O divide a reta em duas semi-retas. A que cont em o ponto U e chamada semi-reta positiva; a que n ao cont em U chama-se semi-reta negativa. Desse modo, dado um ponto X sobre a semi-reta positiva, se X coincidir com a origem O , dizemos que a sua abscissa e o n umero zero; se X n ao coincide com a origem e o segmento OU cabe n vezes no segmento OX , dizemos que a abscissa do ponto X e o n umero natural n; se o segmento OX e comensur avel com o segmento unit ario, de modo que existe um segmento OT que cabe p vezes em OX e q vezes em OU , dizemos que a abscissa de OX e o n umero racional p . q 91

Exemplo: Considere os dois segmentos abaixo:

O 0

T
1 2

U 1

P 2

Q 3

Nesta gura, X e ponto m edio do segmento P Q. Note que OX = 5OT e OU = 2OT . Desse modo, OX 5OT = , OU 2OT onde obtemos que 5 OX = OU. 2 Da segue-se que o comprimento de OX e5 . Logo, a abcissa de X e o n umero racional 5/2. 2 Se o segmento OX n ao for comensur avel com o segmento unit ario, dizemos que a abscissa de X e um n umero irracional. Dessa maneira, ca-se estabelecida uma correspond encia biun voca entre os n umeros reais positivos e os pontos da semi-reta positiva. Analogamente, podemos estabelecer uma correspond encia biun voca entre os pontos da semi-reta negativa e os n umeros reais negativos. Atividade: Determine seis n umeros entre os n umeros: (a) 0, 5 e 0, 7; (b)
3 4 4 e 3 .

8.6.2

Intervalos

Um intervalo fechado da reta e qualquer subconjunto da reta do tipo [a, b] = {x R | a x b} Um intervalo aberto da reta e qualquer subconjunto da reta do seguinte tipo: (a, b) = {x R | a < x < b} , 92

(, b) = {x R | x < b} , (a, +) = {x R | x > a} , (, ) = R. Um intervalo da reta fechado ` a esquerda e um subconjunto da reta do seguinte tipo: [a, b) = {x R | a x < b} , [a, +) = {x R | x a} . Um intervalo da reta fechado ` a direita e um subconjunto da reta do do seguinte tipo: (a, b] = {x R | a < x b} ,

(, b] = {x R | x b} . Situa c ao-problema: Determine os seguintes subconjuntos da reta: (a) (1, 2) [0, 4] (b) (4, 3] [0, ) Solu c ao: (a) Sabemos que (1, 2) = {x R | 1 < x < 2} e [0, 4] =.{x R | 0 x 4} Donde, a interse ca o desses dois intervalos, a qual e o conjunto de todos os n umeros que est ao simultaneamente em ambos os conjuntos, e dada por (1, 2) [0, 4] = {x R | 0 x < 2} Geometricamente, podemos representar esses intervalos do seguinte modo: (1, 2) : [0, 4] :

1 0 93

2 4

(1, 2) [0, 4] = [0, 2) :

(b)Sabemos que (4, 3] = {x R | 4 < x 3} e [0, ) = {x R | x 0}. Ent ao, a uni ao desses dois intervalos, a qual e o conjunto de todos os n umeros que est ao em qualquer um dos dois conjuntos, e dada por (4, 3] [0, ) = {x R | x > 4} Geometricamente, podemos representar esses intervalos do seguinte modo: (4, 3] : [0, +) : (4, 3] [0, +) := (4, +) :

4 0 4

Atividade:Determine os seguintes subconjuntos da reta: (a) (2, 1] [1, 4) ; (b) (, 3) [1, +) .

8.6.3

Valor absoluto ou m odulo de um n umero real

Desse modo, |3| = 3, |7| = (7) , 2 = 2 = 2. Observa c ao:

Seja x um n umero real. O valor absoluto de x e denido como sendo o n umero real x, se x 0 |x| = x, se x < 0

Geometricamente, |x| e a dist ancia da origem O ao ponto X determinado por x sobre a reta real, conforme mostra a gura: O 1 0 94 U 1 |x| X x

Se x = a e x = b s ao as abscissas de dois pontos A e B da reta real, respectivamente, ent ao |b a| e igual a dist ancia entre os pontos A e B , conforme mostra a gura: A |b a| dist ancia entre A e B Situa c ao-problema: Determine a dist ancia entre os pontos de abscissa: a) x 4 e x = 7; b) x = 3 e x = 7; c) x = 2 e x = 4. B

8.7

Produto cartesiano de conjuntos

Terminaremos esta primeira aula, apresentando a no ca o de produto cartesiano de dois conjuntos. Essa no ca o e muito importante para a compreens ao de subconjuntos do plano, destacadamente para a visualiza ca o dos gr acos das fun co es, assunto que ser a abordado nas aulas seguintes. Sejam A e B dois conjuntos. O produto cartesiano de A e B e o conjunto A B de todos os pares de elementos (a, b) nos quais o primeiro elemento e um elemento de A e o segundo elemento e um elemento de B , ou seja, A B = {(a, b) | a A e b B } Exemplo: Sejam A = {1, 2, 3} e B = {2, 4}. O produto cartesiano de A e B e dado por A B = {(a, b) | a {1, 2, 3} e b {2, 4}} Ent ao, A B = {(1, 2) , (1, 4) , (2, 2) , (2, 4) , (3, 2) , (3, 4)} Uma forma instrutiva de visualizar o produto cartesiano e dispondo-se o conjunto A em uma reta horizontal e o conjunto B numa reta vertical, do seguinte modo: 95

0 0 1 2 3

Atividade proposta: Determine o produto cartesiano dos conjuntos A = {2, 5} e B = {2, 4, 6} e represente-o gracamente como um subconjunto do plano. Observa c ao: Se o conjunto A tem m objetos e o conjunto B tem n objetos, ent ao o conjunto A B possui m n elementos. Exemplo: Seja A o intervalo aberto (1, 3) e B o intervalo fechado [2, 3]. Ent ao o produto cartesiano A B = {(a, b) | 1 < x < 3 e 2 b 3} . Podemos visualizar esse produto cartesiano da seguinte maneira: 3 2

Atividade: Seja A o intervalo aberto (3, 2) e B o intervalo fechado a ` esquerda [1, 5]. Determine o produto cartesiano A B e represente-o gracamente como um subconjunto do plano. Observa c ao: Se o conjunto A ou o conjunto B for innito, o produto cartesiano deles tamb em ser a innito. 96

Aula 9 Fun c ao Am
9.1 Conceitos de Fun c ao e de Fun c ao Am

Objetivos
1. Denir e exemplicar fun ca o; 2. Identicar o dom nio e a imagem de fun co es; 3. Esbo car os gr acos de fun co es elementares; 4. Denir e identicar uma fun ca o am; 5. Esbo car o gr aco de uma fun ca o am; 6. Reconhecer uma fun ca o atrav es do gr aco; 7. Estudar o crescimento/decrescimento de uma fun ca o am; 8. Estudar o sinal da fun ca o am; 9. Resolver inequa co es do 10 grau; 10. Resolver problemas que envolvem fun co es am.

96

9.2

Deni c ao de Fun c ao e Exemplos

Um dos mais importantes conceitos do Ensino B asico e o de fun ca o, pois a partir deste conceito conseguimos desenvolver praticamente todos os demais conte udos do Ensino M edio. Tal conceito est a presente em v arias situa co es do nosso dia-a-dia tendo em vista que sempre que estamos com duas grandezas onde uma varia em fun ca o da outra podemos pensar em modelar (equacionar) este problema atrav es de uma fun ca o, de uma lei matem atica que represente tal situa ca o. Formalmente falando temos a seguinte deni ca o: Deni c ao 1. Sejam A e B dois conjuntos n ao vazios. Uma correspond encia f entre A e B que associa a cada elemento x A um u nico elemento y B e chamada de fun c ao. Quando a correspond encia f de A em B e uma fun ca o representamos em s mbolos: f: A x B y = f (x).

Neste caso, o s mbolo y = f (x) nos diz que y e uma fun ca o que depende de x .A letra x e chamada vari avel independente e a letra y que e a vari avel dependente e chamada de fun ca o. Vale observar que trabalhar com x como sendo a vari avel independente e y como sendo a vari avel dependente (fun ca o) e pura op ca o, podemos escolher quaisquer outras duas letras. Exemplo 1. .
a b c d A e f g h B E uma Fun ca o e f g B a b c d e f g

A B N ao e uma Fun ca o 1 0 1 2 A y = x2 0 1 2 4 B uma Fun E ca o

a b c A

N ao e uma Fun ca o

97

Deni c ao 2. Quando f : A B e uma fun ca o o conjunto de partida A e chamado de dom nio (ou campo de exist encia) da fun ca o f e o conjunto de chegada B e chamado de contra-dom nio da fun ca o f. Escrevemos D (f ) = A e CD (f ) = B para representar o dom nio e o contradom nio de f, respectivamente. Em geral, uma fun ca o e dada por uma lei matem atica que precisamos determinar. Vejamos alguns exemplos. Exemplo 2. Considere as seguintes fun c oes: (i) A fun ca o que fornece a diagonal d de um quadrado em fun ca o do lado l e dada por d (l) = l 2. (ii) A fun ca o que fornece a a rea A de um quadrado em fun ca o do lado l e dada por A (l) = l2 . (iii) A fun ca o que fornece o custo C de uma corrida de T axi cuja bandeirada custa R$ 3, 00 e por cada quil ometro rodado R$ 2, 50 e dada por C (x) = 2, 50 x + 3, 00. (iv) A fun ca o que fornece a velocidade v de uma part cula em Movimento Uniformemente Variado (MUV) com velocidade inicial v0 e acelera ca o a e dada por v (t) = v0 + at. (v) A fun ca o que fornece a posi ca o S de uma part cula em Movimento Uniformemente Variado (MUV) com posi ca o inicial S0 , velocidade inicial v0 e acelera ca o a e dada por
1 2 S (t) = S0 + v0 t + 2 at .

O dom nio ou campo de exist encia de uma fun ca o e o maior conjunto dos valores da vari avel independente para o qual a lei matem atica faz sentido, isto e, a lei est a bem denida. Nos exemplos acima, cada uma das leis faz sentido para todos os valores reais de l, x e t. No entanto, nestes exemplos, s o faz sentido trabalhar com valores n ao-negativos de l, x e t pois estamos tratando de medidas. Na maioria dos exemplos que vamos tratar a seguir o dom nio de cada fun ca o e sempre um subconjunto dos n umeros reais, isto e, D (f ) R. Exemplo 3. Determine o dom nio de cada uma das fun c oes abaixo: 98

(i) f (x) = 3x 1. Claramente, esta fun ca o est a bem denida (existe) para qualquer valor real de x. Portanto, D (f ) = R. (ii) f (x) = x2 2x + 3. Claramente, esta fun ca o tamb em est a bem denida (existe) para qualquer valor real de x. Portanto, D (f ) = R. (iii) g (x) = x. Como bem sabemos n ao existe raiz quadrada real de um n umero negativo.

Portanto, D (f ) = R+ = {x R; x 0} . (iv) h (t) = 1 .Como bem sabemos n ao existe divis ao por zero, isto e, o denominador de t uma fra ca o nunca pode ser zero. Portanto, D (f ) = R = {t R; t = 0} .

(v) p (x) =

x3 + x2 + x 1 . Pelo mesmo argumento do item anterior n ao e permitido x2 1 que o denominador de uma fra ca o seja zero. O numerador pode ser qualquer valor. Assim, devemos ter x2 1 = 0, isto e, x = 1. Verique isso! D (f ) = {x R; x = 1} . Logo

Exerc cio 1. Determine o dom nio das seguintes fun co es: (i) f (x) = ax + b, onde a, b R e a = 0, (ii) g (x) = ax2 + bx + c, onde a, b, c R e a = 0, (iii) h (t) = t 2, 2y + 1 (iv) p (y ) = , y1 z+1 . (v) q (z ) = 2 z 4

9.3

Gr aco de uma Fun c ao

De posse do signicado de fun ca o e de dom nio, passamos agora a construir gr acos de algumas fun co es elementares. Para tanto, vamos utilizar uma tabela de valores. Nessa tabela, atribuimos valores a vari avel independente x e obtemos os correspondentes valores da fun ca o y = f (x) . Em seguida, representamos cada um dos pontos obtidos num mesmo plano cartesiano. 99

Exemplo 4. D ( f ) = R.

Construa o gr aco da fun c ao f (x) = x + 1. Inicialmente notamos que

Em seguida, vamos considerar a tabela: x y = x+1 1 0


1 2 1 2

0 1

1 3 4 3

1 2

Localizando esses pontos num mesmo plano cartesiano notamos que os mesmos est ao alinhados. Ligando-os, percebemos que estes pontos formam uma reta. y 3 2 1 2 1 1 2 1 2 3 4 x

Exemplo 5. Construa o gr aco da fun c ao f (x) = x2 . Note que D (f ) = R. Vamos considerar a seguinte tabela: x y = x2 2 2 1 1 1 2
1 4

0 0

1 2 1 4

1 1

Localizando esses pontos num mesmo plano cartesiano percebemos que os mesmos n ao est ao alinhados. Ligando-os vamos formar uma curva aberta (par abola), que tem como ponto mais baixo a origem O (0, 0) . Al em disso, o gr aco desta fun ca o e sim etrico com rela ca o ao eixo dos y.

100

y 2 2 1 1 2 2x

1 O (0, 0)

Exemplo 6. Construa o gr aco da fun c ao f (x) = x3 . Note que D (f ) = R. Vamos considerar a seguinte tabela: x y = x3 1 1
1 2

1 3
1 27

0 0

1 3 1 27

1 2 1 8

1 1

1 8

Localizando esses pontos num mesmo plano cartesiano notamos que os mesmos n ao est ao alinhados. Ligando-os percebemos que vamos formar uma curva que passa pela origem e e sim etrica com rela ca o a origem. y 3 2 1 1 1 2 3 3 1 3 2 x

Deni c ao 3. O subconjunto do contradom nio de f, formado pelos elementos y que est ao associados aos valores de x, e chamado conjunto imagem da fun ca o y = f (x) e denotado por Im (f ) .

101

Vale observar que o conjunto imagem da fun ca o f e um subconjunto do contradom nio, isto e, Im (f ) CD (f ) . Desta forma, a imagem das fun co es dos exemplos 3 e 5 e o conjunto dos n umeros reais (Im (f ) = R) e do exemplo 4 o conjunto dos n umeros reais n ao negativos (Im (f ) = R+ ) . Exerc cio 2. Dadas as fun co es abaixo, identique o dom nio e a imagem de cada uma delas e em seguida construa o gr aco. (i) f (x) = x 1, (ii) f (x) = 2x + 1, (iii) f (x) = x2 1, (iv) f (x) = x, (v) f (x) = x3 1. Sugest ao: Utilize tabelas semelhantes as dos exemplos anteriores. Deni c ao 4. Uma fun ca o f que a cada x R associa sempre um valor constante c e chamada fun c ao constante. Em s mbolos, f: R x R f (x) = c

e chamada fun ca o constante. Observe que o dom nio desta fun ca o e sempre o conjunto dos n umeros reais, isto e, D (f ) = R. J a sua imagem e o conjunto unit ario formado pelo valor real c, isto e, Im (f ) = {c} . O gr aco desta fun ca o e uma reta horizontal (paralela ao eixo dos x) passando pela ordenada c (y = c) . Exemplo 7. Construir o gr aco de f (x) = 5. De acordo com a deni c ao anterior, qualquer que seja x R temos f (x) = 5. Desta forma temos o seguinte esbo co:

102

y 5 y=5

Exerc cio 3. Esboce o gr aco das fun co es constantes. Em seguida, identique o dom nio e a imagem de cada uma delas. (i) f (x) = 2, (ii) f (x) = 2, (iii) f (x) = 0, (iv) f (x) = .

9.4

Fun c ao Am

Passemos agora ao estudo de um tipo especial de fun ca o: a fun ca o am. Existem v arios modelos na natureza que podem ser representados por este tipo de fun ca o ou de seus casos particulares. Deni c ao 5. Uma fun ca o f de R em R dada por f (x) = ax + b onde a e b s ao n umeros reais e a = 0 e chamada fun c ao am. Em s mbolos, f: R x R f (x) = ax + b

e uma fun ca o am para a = 0. Da deni ca o ca claro que o dom nio da fun ca o am eo conjunto dos n umeros reais, isto e, D (f ) = R. Exemplo 8. Identique os coecientes a e b na fun c oes am abaixo. (i) f (x) = 3x 1, a = 3 e b = 1. 103

3 , a = 1 e b = 2 . (ii) f (x) = x + 3 2

(iii) f (x) =

x 3

+ , a =

1 3

e b = .

(iv) f (x) = x, a = 1 e b = 0. (v) f (x) = 3x, a = 3 e b = 0. Estas duas u ltimas fun co es s ao casos particulares da fun ca o am. A fun ca o do item (iv ) e chamada fun c ao identidade e a fun ca o do item (v ) e chamada fun c ao linear. Mais precisamente, qualquer fun ca o real do tipo f (x) = ax, com a = 0 e uma fun ca o linear. O nome linear decorre do fato de a fun ca o f (x) = ax ser bem comportada com rela ca o a ` soma de objetos x e multiplica ca o de um objeto x por um n umero real k, no seguinte sentido: 1. f (x1 + x2 ) = a (x1 + x2 ) = ax1 + ax2 = f (x1 ) + f (x2 ) , x1 , x2 R. 2. f (kx) = a (kx) = k (ax) = k f (x) , x R. Estruturas que possuem comportamento linear s ao as mais cobi cadas na a rea de matem atica.

9.5

Gr aco

No que faremos a seguir a intui ca o nos diz que o gr aco de uma fun ca o, f (x) = ax + b (a = 0) , e sempre uma reta. - Este resultado ser a comprovado posteriormente. Passamos agora a exibir exemplos de gr acos destas fun co es. Exemplo 9. Construa o gr aco da fun c ao f (x) = 3x 1. Utilizando a tabela abaixo e localizando os pontos no plano cartesiano obtemos o gr aco.

x y = 3x 1

1 1 3

0 1

1 3

2 5

104

Observe que o gr aco dessa fun ca o corta o eixo dos x no ponto P no ponto Q (0, 1) .

1 ,0 3

e o eixo dos y

3 . Utilizando a tabela abaixo e localizando Exemplo 10. Idem para a fun c ao f (x) = x + 2

os pontos no plano cartesiano obtemos o gr aco.

x y = x + y 3 2 1 2 1 1 0
3 2

1
5 2

1 2

0
3 2

3 2

2
1 2

y = x +

3 2

Observe que o gr aco dessa fun ca o corta o eixo dos x no ponto P


3 no ponto Q 0, 2 .

3 ,0 2

e o eixo dos y

Exemplo 11. Idem para a fun c ao f (x) = x. Utilizando a tabela abaixo e localizando os pontos no plano cartesiano obtemos o gr aco.

1 1 2

0 0

3 2 3 2

2 2

y = x 1 1 2

105

y 3 y=x 2 1 2 1 1 0 1 2 x

Note que o gr aco dessa fun ca o e a bissetriz do primeiro e terceiro quadrantes, portanto passa pela origem. Exemplo 12. Idem para a fun c ao f (x) = 3x. Utilizando a tabela abaixo e localizando os pontos no plano cartesiano obtemos o gr aco.

1 y 3

1 2 3 2

0 0

3 2 9 2

2 6

y = 3x 3

y = 3x 2 1 1 1 2 0 1 x

Note que o gr aco dessa fun ca o passa pela origem. Dos exemplos acima deduzimos as seguintes propriedades: P1 O gr aco da fun ca o f (x) = ax + b (a = 0) e sempre uma reta. 106

P2 Esta reta sempre corta o eixo dos y no ponto Q (0, b) quando b = 0 pois f (0) = a 0+ b = b. Veja o exemplo 8 (i) e (ii) . P3 O gr aco da fun ca o f (x) = ax e uma reta que passa pela origem. Veja o exemplo 8 (iv ) e (v ) . Exerc cio 4. Escolha tabelas adequadas e construa o gr aco das fun co es abaixo: (i) f (x) = x 1, (ii) f (x) = x + 1, , (iii) f (x) = x 3 (iv) f (x) = x, (v) f (x) = 2x. Do exposto acima, deduzimos que o conjunto imagem da fun ca o f (x) = ax + b (a = 0) e o conjunto dos n umeros reais, isto e, Im (f ) = R. Para se convencer disso, basta ver que dado qualquer y R escolhendo x =
y b a

obtemos f (x) = a

y b a

+ b = y b + b = y.

Vale observar ainda que o gr aco da fun ca o f (x) = ax + b (a = 0) tamb em corta o eixo dos x. O ponto onde isso ocorre tem ordenada y nula. Portanto, fazendo y = 0 e lembrando y = f (x) obtemos ax + b = 0 e a u nica solu ca o dessa equa ca o ex= O valor x =
b a b . a

e chamado zero ou raiz da fun ca o am.

De posse dessas informa co es e sabendo que o gr aco da fun ca o am e sempre uma reta basta escolhermos os dois pontos de intersec ca o para esbo carmos com eci encia seu gr aco. Na pr atica, os pontos P
b a

e Q (0, b) s ao os mais adequados para desenhar o gr aco da

fun ca o am. Tais pontos formam a tabela x y = ax + b


b a

0 b

Exemplo 13. Esboce o gr aco de f (x) = x + 1. Vamos recorrer a tabela dada acima x y = x+1 107 1 0 0 1

y 3 y = x+1 2 1 1 1 2 0 1 x

Exerc cio 5. Esboce o gr aco das fun co es abaixo utilizando apenas os dois pontos de intersec ca o com os eixos coordenados. (i) f (x) = x + 1, (ii) f (x) = x + 2, (iii) f (x) = 3x 1, (iv) f (x) = x + 1, 2 (v) f (x) = 2x. CURIOSIDADE: Reconhecimento de uma fun c ao atrav es do gr aco. Dado um gr aco (ou curva) qualquer no plano cartesiano para saber se o mesmo representa uma fun ca o y = f (x) tra camos retas verticais (paralelas ao eixo dos y ). Se todas estas retas tra cadas cortam o gr aco (ou curva) num u nico ponto ent ao o gr aco em quest ao e de uma fun ca o. Teste isso com os exemplos e exerc cios apresentados acima.

9.6

Crescimento e Decrescimento da Fun c ao Am

Na fun ca o am, f (x) = ax + b (a = 0) , o valor a e chamado coeciente angular e o valor b e chamado coeciente linear. Como bem sabemos o coeciente linear b indica onde o gr aco corta o eixo y. J a o coeciente angular a nos d a informa co es sobre o crescimento da fun ca o, conforme veremos a seguir. Antes por em temos a deni ca o. 108

Deni c ao 6.

Uma fun ca o f e dita crescente quando dados quaisquer x1 e x2

pertencentes ao seu dom nio, com x1 < x2 , tivermos y1 = f (x1 ) < y2 = f (x2 ) . Ou seja, quando os valores de x crescem os valores de y tamb em crescem. Se ocorrer y1 = f (x1 ) > y2 = f (x2 ) para x1 < x2 a fun ca o f e dita decrescente. Exerc cio 6. Verique que a fun ca o f (x) = x + 1 e crescente e que a fun ca o f (x) = 2x + 1 e decrescente. Sugest ao: Fa ca uma tabelinha com dois valores quaisquer de x e investigue o comportamento de y. Propriedade. A fun ca o f (x) = ax + b (a = 0) e crescente quando a > 0 e decrescente quando a < 0. Demonstra c ao. (Vamos fazer apenas a demonstra ca o do primeiro caso, e o segundo, ca como exerc cio.): Seja f (x) = ax + b, com a > 0. Vamos mostrar que f e crescente. Para tanto sejam x1 e x2 pertencentes ao dom nio de f com x1 < x2 e vamos mostrar que f (x1 ) < f (x2 ) , ou, equivalentemente, f (x1 ) f (x2 ) < 0. De fato, basta ver que f (x1 ) = ax1 + b e f (x2 ) = ax2 + b. Portanto, f (x1 ) f (x2 ) = (ax1 + b) (ax2 + b) = a (x1 x2 ) < 0, pois a > 0 e x1 x2 < 0. Exemplo 14. A fun c ao f (x) = 3x + 1 e crescente pois a = 3 > 0 e a fun c ao f (x) = 2x e decrescente pois a = 2 < 0.

9.7

Estudo do Sinal e Inequa c oes

Estudar o sinal da fun ca o f (x) = ax + b (a = 0) signica descobrir os intervalos onde f (x) = 0, f (x) > 0 e f (x) < 0. Nesse estudo, o conhecimento do zero da fun ca o e o sinal do coeciente angular a s ao decisivos. Vamos dividir nosso estudo em dois casos: 10 Caso: a > 0.

109

Neste caso, b e a b f (x) < 0 ax + b < 0 ax < b x < . a f (x) > 0 ax + b > 0 ax > b x > Isto signica que, quando a > 0, a fun ca o e positiva a ` direita de x = esquerda de x =
b . a b a

e negativa a `

+++ b/a

20 Caso: a < 0. Neste caso, f (x) > 0 ax + b > 0 ax > b x < b e a b f (x) < 0 ax + b < 0 ax < b x > . a
b a

Isto signica que, quando a < 0, a fun ca o e positiva a ` esquerda de x = direita de x =


b . a

e negativa a `

+++ b/a

Exemplo 15. Dada a fun c ao f (x) = 3x 1, temos que a = 3 < 0 e que x = Portanto, f (x) > 0 para x >
1 3

1 3

e seu zero.

e f (x) < 0 para x < 1 . Represente gracamente! 3

Exerc cio 7. Estude o sinal da fun ca o f (x) = 2x + 3. Para concluir nosso estudo preliminar sobre a fun ca o am vamos resolver algumas inequa co es do primeiro grau, as quais dependem essencialmente do estudo do sinal da fun ca o 110

am e conseq uentemente das opera co es com intervalos. Nosso estudo ser a apresentado atrav es de exemplos.

Exemplo 16. Resolva a inequa c ao simult anea x + 1 < 2x 1 < x + 8. Solu c ao: Vamos separar a desigualdade dupla e resolver cada inequa ca o. Temos que (1) x + 1 < 2x 1 (2) 2x 1 < x + 8. Seja S1 o conjunto solu ca o da inequa ca o (1). Resolvendo (1) temos que x + 1 < 2x 1 x 2x < 1 1 x < 2 ? ? x > 2. Logo, S1 = {x R/ x > 2 }. Seja S2 o conjunto solu ca o da inequa ca o (2). Resolvendo (2) temos que 2x 1 < x + 8 2x + x < 8 + 1 3x < 9 x < 3. Logo, S2 = {x R/ x < 3}. Agora, representando esses dois intervalos na reta real e tomando a intersec ca o (por qu e ?), chegamos a solu ca o S = S1 S2 = {x R; 2 < x < 3} = (2, 3) .

S2 : x < 3 : S1 : x > 2 : S:2<x<3: 2 2

Exerc cio 8. Resolva a inequa ca o simult anea 2x 10 x + 5 < 5x 7. Exemplo 17. Resolva a inequa c ao produto (x + 1) (2x + 1) 0. 111

Solu c ao: Tal inequa ca o e chamada produto porque temos um produto entre duas fun co es. Vamos chamar a primeira fun ca o de f e a segunda de g.Em seguida, fazemos o estudo de sinal de cada uma delas localizamos na reta real e numa terceira reta real fazemos o produto dos sinais, nos intervalos determinados pelos zeros de cada fun ca o. Temos ent ao. 1. f (x) = x + 1 temos que a = 1 > 0 e x = 1 e o zero de f . Portanto, f (x) 0 para x 1 e f (x) 0 para x 1. 2. Para g (x) = 2x + 1 temos que a = 2 < 0 e x =
1 e g (x) 0 para x 2 . g (x) 0 para x 1 2 1 2

e o seu zero de g . Portanto,

Ap os fazer o produto dos sinais obtemos o conjunto solu ca o S = x R; 1 x Representando na reta temos: + + 1
|

1 2

f (x) : g (x) : f (x) g (x) :

+ 1/2
|

+ +

1/2

Vale observar que como as regras de sinais s ao as mesmas para produto e quociente as inequa co es quociente podem ser resolvidas de maneira an aloga as inequa co es produto, tomando apenas o cuidado que o denominador n ao pode ser zero. Exerc cio 9. Resolva as inequa co es abaixo: (i) (x + 3) (4x + 1) 0, x+3 (ii) 0. 4x + 1

112

Aula 10 Fun co es Quadr aticas


Objetivos
1. Denir e identicar uma fun ca o quadr atica 2. Esbo car com eci encia o gr aco de fun co es quadr aticas 3. Investigar as principais propriedades das fun co es quadr aticas (dom nio, imagem, gr aco, v ertice, valor extremo, etc) 4. Estudar o sinal da fun ca o quadr atica 5. Resolver inequa co es quadr aticas 6. Resolver problemas que envolvem fun co es quadr aticas.

10.1

Deni c oes e Exemplos

Denio 10.1. Uma aplica c ao f de R em R recebe o nome de fun c ao quadr atica, ou do segundo grau, quando associa a cada x R o elemento (ax2 + bx + c) R, onde a = 0. Em s mbolos, f: R R x y = f (x) = ax2 + bx + c (a = 0) .

Os n umeros reais a, b e c s ao chamados coecientes da fun c ao quadr atica. Por que precisamos ter a = 0? 112

Exemplo 1 Identique os coecientes das seguintes fun co es quadr aticas: (i) f (x) = x2 3x + 2, onde a = 1, b = 3 e c = 2 (ii) f (x) = x2 4, onde a = 1, b = 0 e c = 4 (iii) f (x) = 2x2 + 5x, onde a = 2, b = 5 e c = 0 (iv) f (x) = 3x2 , onde a = 3, b = 0 e c = 0. Da deni ca o acima e claro que o dom nio ou campo de exist encia da fun ca o f (x) = ax2 + bx + c e o conjunto dos n umeros reais, escrevemos D (f ) = R. Para se convencer disso basta ver que atribuindo qualquer valor real r a vari avel x temos em correspond encia o valor f (r ) = (ar 2 + br + c) R. Pense nisso! Passemos agora a desenhar alguns gr acos de fun co es quadr aticas, nos exemplos a seguir, com base na tabela dada em cada exemplo. Exemplo 2 y = x2 . x 2 y = x2 4 1 1 0 1 2 0 1 4

5 4 3 2 1 2 1 1 O (0, 0) 1 2

113

Exemplo 3 y = x2 + 1. x 3 2 3 1 0 0 1 1 0 2 3 3 8

y = x2 + 1 8

1 O (0, 0) 3 2 1 1 2 3 4 5 6 7 8 1 2 3

Exemplo 4 y = x2 2x. x 2 1 3 8 7 6 5 4 3 2 1
O (0, 0)

0 0

1 1

2 3 4 0 3 8

y = x2 2x 8

3 2 1 1 Exemplo 5 y = x2 3x + 2.

114

x y = x2 3x + 2 8 7 6 5 4 3 2 1

1 6

0 1 2 2 0 0

3 4 2 6

O (0, 0)

1 1

Exemplo 6 y = x2 2x + 3. x 1 0 1 3 2 2 3 3 6

y = x2 2x + 3 6

115

7 6 5 4 3 2 1
O (0, 0)

1 Exemplo 7 y = x2 + 2x 1. x 1

0 1

1 0

2 1

3 4

y = x2 + 2x 1 4 2 1 O (0, 0) 1 1 2 3 4 1

Dos exemplos acima vimos que o gr aco da fun ca o quadr atica e uma curva aberta. Tal curva e chamada de par abola e ser a estudada com mais detalhes no curso de Geometria Anal tica. Tamb em podemos observar (deduzir) desses exemplos as seguintes propriedades para a fun ca o f (x) = ax2 + bx + c: 116

Exerc cio 1. Construa o gr aco das fun co es abaixo tomando como refer encia as tabelas dos exemplos 2 a 7 acima. Em seguida discuta (identique) as propriedades P1-P4. (i) y = x2 ; (ii) y = x2 4; (iii) y = x2 + 3x; (iv) y = x2 + x 6; (v) y = x2 2x + 2; (vi) y = x2 2x + 1. Do exposto acima percebemos que o gr aco da fun ca o y = ax2 + bx + c intercepta o eixo Ox quando a equa ca o ax2 + bx + c = 0 possui ra zes reais, veja os exemplos (2)-(5) e (7). Al em disso, em alguns casos a par abola toca o eixo Ox num u nico ponto, veja os exemplos (2) e (7) . No exemplo (6) a par abola n ao toca o eixo Ox.

10.2

Forma Can onica e Zeros

A constru ca o do gr aco da fun ca o y = ax2 + bx + c usando apenas tabelas, como acima, e bastante imprecisa pois atribu mos apenas valores inteiros para a vari avel x. Como bem sabemos, as ra zes da equa ca o ax2 + bx + c = 0 e o maior ou menor valor assumido pela ordenada y pode n ao ser um n umero inteiro. Nossos pr oximos desaos consistem em formalizar as propriedades listadas acima e explorar novas propriedades da fun ca o quadr atica. Levando em conta nosso conhecimento sobre a equa c ao quadr atica ax2 + bx + c = 0 (a = 0) , estudada em Algebra Elementar, vamos utilizar algumas dedu co es alg ebricas para justicar as propriedades acima. Inicialmente, observe que podemos reescrever a fun ca o f (x) = ax2 + bx + c = 0 (a = 0) na forma can onica. A saber,

117

f (x) = ax2 + bx + c b c = a x2 + x + a a b b2 = a x2 + x + 2 a 4a =a b x+ 2a
2

b2 c + 2 4a a .

b2 4ac 4a2

Portanto, para obter os zeros ou ra zes da fun ca o f (x) = ax2 + bx + c devemos fazer f (x) = 0 e com isso precisamos resolver a equa ca o quadr atica ax2 + bx + c = 0. Ou seja, devemos resolver a equa ca o a b x+ 2a
b 2 2a 2

b2 4ac 4a2

= 0.
b 2 2a

Como a = 0 devemos ter x + isso x +


b 2a

b2 4ac 4a2

= 0. Ou ainda, x +

b2 4ac 4a2

e com

b2 4ac . 4a2

Logo, b x= 2a b2 4ac b b2 4ac = . 4a2 2a

Conforme j a foi discutido no estudo da equa ca o quadr atica, o termo b2 4ac e o famoso discriminante, da equa ca o, que e representado pelo letra grega delta (mai uscula), isto e, := b2 4ac. As ra zes da equa ca o ax2 + bx + c = 0, quando existem, s ao dadas por x=
b b2 4ac 2a b . 2a

A discuss ao sobre a quantidade de ra zes da equa ca o quadr atica e suas propriedades j a foi apresentada na disciplina Algebra Elementar. Contudo, relembramos que: 1. x1 = Quando > 0 esta equa ca o possui duas ra zes reais e distintas, dadas por
b 2a b + . 2a b 2a

e x2 =

2. Quando = 0 esta equa ca o possui uma u nica raiz real, dada por x1 = x2 = qu e?). 3. Quando < 0 esta equa ca o n ao possui ra zes reais. Por qu e?

(por

118

Exemplo 8. Construa o gr aco da fun ca o y = x2 4x + 3 e identique suas intersec co es com os eixos coordenados. Solu c ao: Vamos utilizar a tabela x y = x2 4x + 3 1 8 0 1 2 3 0 4 5 3 8

3 0 1

Observe que as ra zes ou zeros da fun ca o y = x2 4x + 3 s ao exatamente as ra zes da equa ca o x2 4x + 3 = 0, que s ao x1 = 1 e x1 = 3. Portanto a par abola intercepta o eixo das abscissas nos pontos A (1, 0) e B (3, 0) . Al em disso, como f (0) = 3 temos que a par abola intercepta o eixo das ordenadas no ponto P (0, 3) . Com estas informa co es e o fato que a concavidade e voltada para cima (pois a = 1 > 0) temos o esbo co do gr aco. 6 5 4 3 2 1 O (0, 0) 1 1 1 2 3 4

Exerc cio 2. Esboce o gr aco das fun co es abaixo e identique suas intersec co es com os eixos coordenados. (i) f (x) = x2 5x + 6; (ii) f (x) = x2 + 4x 4; (iii) f (x) = x2 2x + 1; (iv) f (x) = x2 + 4x 5. Exemplo 9. Determine os valores de m de sorte que a fun ca o f (x) = mx2 +(2m 1) x+ 119

(m 2) tenha dois zeros reais e distintos. Solu c ao: Como bem sabemos a fun ca o acima possui dois zeros reais e distintos quando o discriminante da equa ca o f (x) = 0 mx2 + (2m 1) x + (m 2) = 0 e positivo. Nesta equa ca o temos a = m, b = (2m 1) e c = m 2. Portanto := b2 4ac = (2m 1)2 4 m (m 2) = 4m + 1. Agora , = 4m + 1 > 0 implica 4m > 1.Ou seja, devemos ter m > 1/4. Exerc cio 3. Determine os valores de m de sorte que a fun ca o f (x) = mx2 + (m + 1) x + (m + 1) tenha um zero real duplo. Exerc cio 4. Determine os valores de m de sorte que a fun ca o f (x) = (m + 1) x2 + (2m + 3) x + (m 1) n ao tenha zeros reais. Exerc cio 5. Dada a equa ca o quadr atica 2x2 5x 1 = 0 com ra zes x1 e x2 calcule: (i) x1 + x2 ; (ii) x1 x2 ; (iii) (iv)
1 x1 x1 x2

+ +

1 ; x2 x2 ; x1

(v) (x1 )2 + (x2 )2 .

10.3

Valor M aximio ou M nimo e V ertice

Confome mencionado antes, a fun ca o quadr atica y = ax2 + bx + c (a = 0) assume sempre um valor extremo (valor m aximo ou valor m nimo) o qual depende do sinal do coeciente a. Denio 10.2. Dizemos que o n umero yM Im (f ) e o valor m aximo da fun c ao y = f (x) quando yM y para qualquer y Im (f ). O valor xM D (f ) tal que yM = f (xM ) e chamado ponto de m aximo da fun c ao. 120

Denio 10.3. Dizemos que o n umero ym Im (f ) e o valor m nimo da fun c ao y = f (x) quando ym y para qualquer y Im (f ) . O valor xm D (f ) tal que ym = f (xm ) e chamado ponto de m nimo da fun c ao. Propriedade 1. Quando a < 0 a fun ca o quadr atica y = ax2 + bx + c admite um valor m aximo y =
4a

em x =

b . 2a

Propriedade 2. Quando a > 0 a fun ca o quadr atica y = ax2 + bx + c admite um valor m nimo y =
4a

em x =

b . 2a

Prova: Vamos provar apenas a Propriedade 1. exerc cio para o leitor, pois e an aloga.

Deixamos a Propriedade 2 como

Consideremos a fun ca o quadr atica na forma can onica y=a Tendo em vista que x +
b 2 2a

x+

b 2a

. ( ) 4a2

0 para qualquer x real e 4 e um valor constante a

para cada fun ca o dada, ent ao y assumir a um valor m aximo quando a < 0 e a diferen ca x+
b 2 2a

4a

for a menor poss vel, isto e, quando x +


b 2a

b 2 2a

= 0, que implica x =

b . 2a

Agora, substituindo x =

em () obtemos b b + 2a 2a
2

y=a

4a2

= a 02

= . 4a2 4a

portanto = (4)2 4.4. (8) = 144. Como a = 4 > 0 a fun ca o admite um valor m nimo. A saber ym =
4a

Exemplo 10. Na fun ca o real f (x) = 4x2 4x 8 temos que a = 4, b = 4 e c = 8,

144 44

= 9. Tal valor m nimo ocorre em xm =

b 2a

(4) 24

=1 . 2

Exerc cio 6. x2 + x + 3 . 4 Denio 10.4. y = ax2 + bx + c.

Obtenha o valor extremo (m aximo ou m nimo) da fun ca o f (x) =

O ponto V

b , 2a 4a

e chamado v ertice da par abola dada pela fun c ao

121

Exemplo 11. Dentre todos os n umeros de reais de soma 8 determine aqueles cujo produto e m aximo. Solu c ao: Indicando por x e z esses n umeros e por y o seu produto, temos:

x + z = 8 e y = x z. Agora, precisamos isolar uma das vari aveis x ou z. Vamos fazer op ca o de tirar o valor de z na primeira equa ca o e substituir na segunda. Portanto, z = 8 x e com isso
b 2a

y = x z = x (8 x) = x2 + 8x, ou seja, f (x) = x2 + 8x. Como a = 1 < 0 esta fun ca o assume um valor m aximo que e atingido no ponto de m aximo x = =
8 2

= 4.

Substituindo este valor em z = 8 x obtemos z = 4. Logo os n umeros procurados s ao 4 e 4. Exerc cio 7. Dentre todos os n umeros x e z tais que 2x + z = 8 determine aqueles cujo produto e m aximo. Exerc cio 8. Dentre todos os ret angulos de per metro 20cm determine o de a rea m axima. Sugest ao: A a rea A e o per metro P de um ret angulo gen erico de dimens oes x e y s ao dados por A = x y e P = 2x + 2y. Exerc cio 9. Determine o valor de m na fun ca o real f (x) = (m 1) x2 +(m + 1) x m de sorte que seu valor m nimo seja 1.

10.4

Conjunto Imagem

Nosso objetivo agora e identicar o conjunto imagem da fun ca o quadr atica y = f (x) = ax2 + bx + c (a = 0) . Utilizando novamente a forma can onica da fun ca o quadr atica temos que f (x) = ax2 + bx + c =a b b + 2a 2a 122
2

, 4a2

ou seja, f (x) = a Observamos agora que considerar: 10 Caso. Quando a > 0, a 20 Caso. Quando a < 0, a Resumindo temos: , x R 4a a < 0 y , x R. 4a a>0y Destas observa co es podemos estabelecer o seguinte resultado. Propriedade 3. O conjunto imagem da fun ca o f (x) = ax2 + bx + c e dado por
b 2a b 2a b 2a

b b + 2a 2a

. 4a

b 2 2a

0 para qualquer x real. Ent ao temos dois casos a + +


b 2 2a b 2 2a

0 e portanto y = a 0 e portanto y = a

b 2a b 2a

+ +

b 2 2a b 2 2a

4a 4a

4 . a 4 . a

Im (f ) = Im (f ) =

4a y R; y 4a y R; y

, para a > 0 e , para a < 0.

Exemplo 12. Obtenha a imagem da fun ca o f : R R dada por f (x) = 2x2 8x + 6. Solu c ao: Na fun ca o f (x) = 2x2 8x + 6, temos a = 2, b = 8 e c = 6. Portanto, = b2 4ac = (8)2 4 2 6 = 16
e com isso 4 = a 16 42

= 2. Como a = 2 > 0, temos pela propriedade acima que

Im (f ) = {y R; y 2} . Exerc cio 10. f (x) =


x 2 3

Obtenha o conjunto imagem da fun ca o f : R R dada por

+ 2x 5 . 3

123

10.5

Eixo de Simetria da Par abola

Pela experi encia adquirida at e agora, no tocante ao esbo co de gr acos, podemos concluir que a par abola possui certas regularidades e uma not avel e a exist encia de um eixo de simetria. A grosso modo falando existe uma reta vertical (perpendicular ao eixo dos x) que divide a par abola ao meio. Tal reta passa sempre pelo v ertice V e portanto e dada pela equa ca o x =
b . 2a b , 2a b , 2a 4a

da par abola

Para provarmos que a par abola tem eixo de simetria na reta x = que dado um ponto A B
b 2a b 2a

devemos mostrar

r, y , com r R, pertencente ao gr aco da fun ca o, existe


b 2a

+ r, y tamb em pertencente ao gr aco da fun ca o. +


b 2 2a

De fato, tomando a fun ca o quadr atica na forma can onica f (x) = a e usando o fato que A
b 2a

4a2

r, y pertence ao gr aco da par abola temos: b r 2a b b r+ 2a 2a


2

y=f =a

4a2

4a2 = a (r )2 2 4a = a (r )2 =a =f Portanto est a provado que B


b 2a

b b +r+ 2a 2a b +r . 2a

4a2

+ r, y tamb em pertence ao gr aco da fun ca o.

10.6

Gr aco

Com base no estudo feito at e agora, das propriedades da fun ca o f (x) = ax2 + bx + c, vamos fazer um resumo com o objetivo de facilicitar o esbo co, com eci encia, da par abola que representa a fun ca o quadr atica y = f (x) . 124

Relembramos, da aula anterior, que para esbo car com eci encia o graco da fun c ao am f (x) = ax + b (a = 0) basta escolhermos dois pontos, pois neste caso o gr aco e uma reta. Al em disso, para simplicar as contas e o desenho, podemos sempre escolher os pontos de intersec ca o desta reta com os eixos coordenados. A saber, A
b ,0 a

e B (0, b) .Temos

ent ao com a tabelinha esperta para a fun ca o f (x) = ax + b (a = 0)

x y = ax + b

0 b

b a

0.

Para a fun ca o quadr atica temos as seguintes propriedades que auxiliam no esbo co do gr aco: I. A concavidade da par abola e voltada par acima quando a > 0 e para baixo quando a < 0. II. A par abola pode ser desenhada com tr es pontos bem escolhidos. Que pontos s ao esses? Resposta: retorno. V
b , 2a 4a

Devemos escolher sempre o v ertice da par abola, que e o ponto de Os outros dois, podem ser quaisquer dois pontos sim etricos do v ertice

da par abola.

III. Na pr atica, podemos sempre trabalhar com a tabela x y = f (x) x1 y1 xv yv x2 y2 .

A escolha dos pontos x1 e x2 dependem fortemente do valor de = b2 4ac. Quando := b2 4ac > 0 escolha x1 e x2 como sendo os zeros da fun ca o f (x) = ax2 + bx + c. Quando = 0 ou < 0 escolha x1 e x2 (x1 < x2 ) como sendo os sim etricos do valor xv =
b ,isto 2a

e, xv =

x 1 +x 2 . 2

Vale observar que quando = 0 o valor xv =

b 2a

exatamente a u nica raiz ou zero da fun ca o. 125

Esta tabela ajuda consideravelmente no esbo co do gr aco pois quando 0, com estas escolhas, estamos identicando as intersec co es do gr aco com o eixo dos x. x y = f (x) x1 0 xv yv x2 0.

Um ponto importante que deve ser escolhido sempre e o ponto P (0, f (0)) , que fornece a intersec ca o do gr aco com o eixo dos y. Exerc cio 12. Repita o exerc cio 2, escolhendo a tabela esperta para cada fun ca o. Em seguida, identique o valor extremo y = f (x) y1 yv y2 . (m aximo ou m nimo) de cada fun ca o, seu v ertice, conjunto imagem e o eixo simetria. Curiosidade: Por que as antenas parab olicas possuem o formato de uma Par abola? E os far ois de carro? Pesquise sobre isso. x x1 xv x2

10.7

Estudo do Sinal e Inequa c oes

Para concluir nossa abordagem sobre a fun ca o quadr atica vamos fazer o estudo do sinal e resolver algumas inequa co es. Inicialmente informamos que estudar o sinal da fun ca o quadr atica f (x) = ax2 + bx + c (a = 0) signica identicar os valores de x R para os quais ocorre: (i) f (x) > 0, (ii) f (x) < 0, (iii) f (x) = 0. Na determina ca o do sinal da fun ca o quadr atica, devemos come car pelo c alculo do discriminante , onde um dos tr es casos distintos podem aparecer: (i) < 0, 126

(ii) = 0, (iii) > 0. 10 Caso: < 0 Se < 0, ent ao > 0 e pela f ormula can onica temos que a f (x) = a
2

b x+ 2a

4a2

e portanto a f (x) > 0, x R, pois a2 > 0, x +

b 2 2a

0 e 4 > 0. Portanto, a fun ca o a2

f (x) = ax2 + bx + c, quando < 0, tem o sinal de a para todo x R, ou melhor: a > 0 f (x) > 0, x R a < 0 f (x) < 0, x R. Exemplo 14. Para a fun ca o f (x) = x2 2x +2 temos que = (2)2 4 1 2 = 4 < 0 e, como a = 1 > 0, conclu mos que f (x) > 0, x R. Exerc cio 13. Estude o sinal da fun ca o f (x) = x2 + x 1. 20 Caso: = 0 Da forma can onica, temos: a f (x) = a pois a2 > 0 e x +
b 2 2a 2

b x+ 2a

0 2 4a

=a

b x+ 2a

0, x R,

0. Isto signica que a fun ca o f (x) = ax2 + bx + c, quando = 0,


b 2a

tem o sinal de a para todo x R com excess ao do valor x = f (x) ,ou melhor:

que e o zero real duplo de

a > 0 f (x) 0, x R a < 0 f (x) 0, x R. 127

Exerc cio 14. Estude o sinal das fun co es f (x) = x2 2x +1 e g (x) = 2x2 +8x 8. 30 Caso: > 0 Neste caso, a fun ca o f (x) = ax2 + bx + c possui dois zeros reais e distintos, x1 e x2 , dados por b x1 = 2a b + x2 = . 2a Com isso, a f (x) = a2 =a
2

x+

b 2a

4a2 b + x 2a

b x 2a

= a2 (x x1 ) (x x2 ) e o estudo do sinal e bem mais interessante. Deixamos como exerc cio para o leitor, vericar que: 1. O sinal de f (x) e o sinal de a para todo x, tal que x < x1 ou x > x2 . Ou seja, eo sinal de a fora do intervalo compreendido entre as ra zes x1 e x2 . 2. O sinal de f (x) e o sinal de a para todo x, tal que x1 < x < x2 , isto e, entre as ra zes x1 e x2 . Exemplo 15. Dada a fun ca o f (x) = x2 x 6 temos que = (1)2 4 1 (6) = 25 > 0. Portanto, f (x) tem dois zeros reais distintos dados por x1 =
x2 = b+ = 1+5 = 3. Como a = 1 > 0, conclu mos que: 2 2a f (x) > 0 para x < 2 ou x > 3 b 2a

15 2

= 2 e

f (x) = 0 para x = 2 ou x = 3 f (x) < 0 para 2 < x < 3.

Exerc cio 15. Estude o sinal da fun ca o f (x) = 2x2 + 3x + 2. 128

Finalmente, vamos concluir nosso estudo preliminar sobre a fun ca o quadr atica resolvendo inequa co es do tipo ax2 + bx + c > 0, ax2 + bx + c < 0, ax2 + bx + c 0 ou ax2 + bx + c 0 (a = 0) , que s ao denominadas inequa co es do segundo grau. Tal estudo depende essencialmente do sinal da fun ca o quadr atica, que zemos acima. A t tulo de ilustra ca o vamos tratar dois exemplos: Exemplo 16. Resolver a inequa ca o x2 2x + 2 > 0. Resolu c ao: Considerando f (x) = x2 2x + 2 temos que a = 1 > 0 e = 4 < 0. Portanto f (x) > 0, x R. Como a inequa ca o de interresse e f (x) > 0, temos que S = R. Exemplo 17. Resolver a inequa ca o produto (x2 x 2) (x2 + 4x 3) > 0 em R. Resolu c ao: Considerando a fun ca o f (x) = x2 x 2 temos que a = 1 > 0, = 9 > 0 e seus zeros s ao 1 e 2. Analogamente, para g (x) = x2 + 4x 3 temos que a = 1 < 0, = 4 > 0 e seus zeros s ao 1 e 3. Agora, colocamos os zeros de cada fun ca o numa reta e fazemos o estudo do sinal. Em seguida, multiplicamos o sinal em cada intervalo determinado pelas ra zes e chegamos ao conjunto solu ca o procurado;

S = {x R; 1 < x < 1 ou 2 < x < 3} . Deixamos como exerc cio para o leitor, fazer o estudo do sinal das fun co es f e g acima. Em seguida, fazer o produto dos sinais em cada intervalo determinado pelas ra zes para conrmar o conjunto solu ca o S acima.

129

Aula 11 Fun c ao Modular


Objetivos
1. Denir o m odulo ou valor absoluto de um n umero real 2. Estabelecer as principais propriedades do m odulo de um n umero real 3. Denir fun ca o modular 4. Identicar as principais propriedades da fun ca o modular (dom nio, imagem, gr aco, etc) 5. Resolver equa co es e inequa co es modulares 6. Estudar a fun ca o c ubica f (x) = x3 (dom nio, imagem, gr aco e propriedades) 1 7. Estudar a fun ca o rec proca f (x) = (dom nio, imagem, gr aco e propriedades) x 8. Estudar a fun ca o m aximo inteiro f (x) = [[x]] 9. Estudar fun co es denidas por v arias senten cas (dom nio, imagem, gr aco e

propriedades)

129

11.1

M odulo ou Valor Absoluto de um N umero Real

Seja x um n umero real qualquer. Denimos o m odulo ou valor absoluto de x, x, se x 0 representado por |x|, como sendo |x| = .Isto signica que: ou x, se x < 0 1. O m odulo de um n umero real n ao negativo e igual ao pr oprio n umero 2. O m odulo de um n umero real negativo e igual ao oposto desse n umero Exemplo 11.1. (i) |5| = 5, pois 5 > 0 (ii) | | = , pois > 0 (iii) |3| = (3) = 3, pois 3 < 0 (iv) 5 = 5 , pois 5 < 0,

(v) |0| = 0, pois 0 0. Dos exemplos acima concluimos que a u nica fun ca o da opera ca o m odulo e transformar um n umero real noutro positivo. Quando este j a e positivo nada mais a fazer.

11.2

Proprieades do M odulo de um N umero Real

Sejam x, y dois n umeros reais. Valem as seguintes propriedades: (i) |x| 0, (ii) |x| = 0 x = 0, (iii) |x y | = |x| |y | , (iv) |x|2 = x2 , 130

(v) x |x| , (vi) |x + y | = |x| + |y | , (vii) |x y | = |x| |y | , (viii) Para a > 0 (a R) , |x| = a x = a, (ix) Para a > 0 (a R) , |x| a a x a, (x) Para a > 0 (a R) , |x| a x a ou x a. Deixamos a verica ca o dessas propriedades como exerc cio para o leitor. Se conven ca da validade das mesmas atrav es de exemplos, casos particulares.

11.3

Fun c ao Modular

Uma aplica ca o f de R em R recebe o nome de fun ca o m odulo ou modular quando a cada x R associa o elemento |x| R. Ou seja, f: R x e chamada fun ca o modular. Utilizando o conceito de m odulo de um n umero real, a fun ca o modular pode tamb em ser denida como x, se x 0 f (x) = . x, se x < 0 R y = |x|

Ou ainda,

Como podemos ver a fun ca o f (x) = |x| e denida por duas leis (senten cas) y = f1 (x) e .y = f2 (x) . Uma delas funciona para x 0 e a outra para x < 0.Portanto, D (f1 ) = R+ e
D (f1 ) = R . Como R+ R = R temos que D (f ) = R.

f1 (x) = x, se x 0 . f (x) = f (x) = x, se x < 0


1

131

Pela deni ca o da fun ca o modular concluimos que seu gr aco e a uni ao de duas semiretas, bissetrizes do primeiro e segundo quadrantes, que se encontram na origem. Fazendo uma simples tabela e respeitando o dom nio de y = f1 (x) e y = f2 (x) temos o gr aco de f (x) = |x| . x y = |x| 4 3 2 y = |x| 1 1 P (0, 0) 1 2 3 1 0 1 0 1 1

Do gr aco de f (x) = | x| temos que y n ao assume valores negativos, portanto Im (f ) = R+ .

Exerc cio 1. Esboce o gr aco da fun ca o f (x) = |3x| . Exemplo 11.2. Esboce o gr aco da fun c ao f (x) = |x + 1| . Solu c ao 11.1. Usando a deni c ao de m odulo temos que + (x + 1) = x + 1, se x + 1 0 x 1 | x + 1| = . (x + 1) = x 1, se x + 1 < 0 x < 1 Portanto, f1 (x) = x + 1, se x 1 f (x) = |x + 1| = . f (x) = x 1, se x < 1
2

Fazendo o gr aco de y = f1 (x) e .y = f2 (x), em seus dom nios, a partir da tabela abaixo temos a reuni ao de duas semi-retas que se encontram no ponto P (1, 0) .Podemos utilizar 132

a tabela x y = | x + 1| 2 1 1 0 0 1

3 y = | x + 1| 2 1 4 3 2 1 P (0, 1)

Exerc cio 2. Esboce o gr aco das fun co es: (i) f (x) = |x 1| , (ii) f (x) = |x 1| + 2. Exemplo 11.3. Esboce o gr aco da fun c ao f (x) = |x| . x

Solu c ao 11.2. Inicialmente observamos que x n ao pode assumir o valor 0 (zero). Portanto D (f ) = R .Usando a deni c ao de m odulo temos que x |x| x = 1, se x > 0 = . x x = 1, se x < 0 x f1 (x) = 1, se x > 0 f (x) = . f (x) = 1, se x < 0
2

Portanto,

Como as fun c oes y = f1 (x) e y = f2 (x) s ao constantes, em seus dom nios, o gr aco de cada uma delas e uma semi-reta paralela ao eixo dos x.Observe que existe um salto entre estas duas semi-retas pois uma est a no primeiro quandrante partindo do ponto P (0, 1) e a outra est a no quarto quadrante partindo do ponto Q (0, 1) . -Os pontos P e Q n ao pertencem ao 133

gr aco pois n ao podemos ter x = 0.Podemos utilizar a tabela x f (x) = |x| y= x 2 1 |x| x 2 1 1 1 1 1 2 1

P (0, 0) 1 2

Exerc cio 3. Esboce o gr aco das fun ca o: |x 1| , para x = 1, (i) f (x) = x1 (ii) f (x) = | |2x 2| 4 | .

11.4

Equa c oes e Inequa c oes Modulares

Para resolver equa co es modulares utilizamos a propriedade (viii) acima, isto e, se a > 0 (a R) , |x| = a x = a. a a R

Exemplo 11.4. Resolva a equa c ao |2x 1| = 3. Solu c ao 11.3. Pela propriedade acima temos que 2x 1 = 3 x = 2 ou | 2x 1| = 3 Logo S = {1, 2} . 2x 1 = 3 x = 1. Exemplo 11.5. Resolva a equa c ao |3x 1| = |2x + 3| .

134

Solu c ao 11.4. Novamente pela propriedade (viii) temos |3x 1| 3x 1 = 2x + 3 x = 4 ou 2 , 4 . Logo S = 5 3x 1 = 2x 3 x = 2 .


5

| 2x + 3|

Exemplo 11.6. . Resolva a equa c ao | x + 1 | = 3 x + 2 .

Solu c ao 11.5. Inicialmente observe que precisamos ter 3x + 2 0 x igualdade fa ca sentido. Agora, usando a propriedade (viii) temos que x + 1 = 3x + 2 x = 1 ou 2 |x + 1| = 3x + 2 x + 1 = 3x 2 x = 3 .
4

2 3

para que a

Observe que x = S=
1 2

3 4

n ao satisfaz pois devemos ter x

2 3

pela condi c ao inicial. Logo

Exerc cio 4. Resolva as equa co es: (i) |4x 5| = 0 (ii) |3x + 2| = |x 1| (iii) |2x 5| = x 1 (iv) |x2 + x 5| = |4x 1| (v) |x2 + 2x 2| = |x2 x 1| . Passamos agora ao estudo das inequa co es modulares. propriedades (ix) e (x) vistas anteriormente, isto e, Tal estudo decorre das

(1) | x| a a x a, para a > 0 (2) |x| a x a ou x a, para a > 0.

a a

a 135

Exemplo 11.7. . Resolva a inequa c ao |2x + 1| < 3 em R. Solu c ao 11.6. Utilizando a parte (1) da propriedade acima temos que |2x + 1| < 3 3 < 2x + 1 < 3 3 1 < 2x < 3 1 4 < 2x < 2 2 < x < 1. Portanto, S = {x R; 2 < x < 1} .Essa inequa c ao modular produz uma inequa c ao simult anea que pode ser resolvida de maneira diferente. Resolve-se cada inequa c ao , representa-se na reta os intervalos correspondentes e em seguida faz-se a intersec c ao dos mesmos. Veja a parte nal de fun c ao am.

136

Aula 12 Fun co es Exponenciais


Iniciaremos esta aula recordando, na se ca o 5.1, alguns fatos b asicos acerca da opera ca o de potencia ca o. Esse conhecimento e essencial para compreendermos as fun co es exponenciais, assunto que ser a abordado na se ca o 5.2. N ao temos a inten ca o de exaurir o tema das fun co es exponenciais e das fun co es logar tmicas (aula 6), mas sim de dar a ele uma primeira aproxima ca o. O leitor interessado em saber mais sobre o tema pode consultar a bibliograa sugerida no m das aulas 5 e 6. Os livros a citados podem ser acessados na biblioteca ou adquiridos no Instituto de Matem atica da UFAL.

12.1

Pot encia de expoente inteiro

Deni c ao: Seja a um n umero real. Suponha que n e um n umero inteiro positivo. Denimos a pot encia de base a e expoente n como sendo o n umero obtido da seguinte maneira: an = a a . . . a (produto de n fatores) Al em disso, se a = 0 e n e um inteiro positivo, denimos an = Exemplos: (a) a4 = a a a a; 138 1 an

(b)

(c) 23

= 3 3; 1 = 3. 2
2

Deni c ao: Para pot encias de expoente zero, denimos a0 = 1 , para a = 0. Observa c ao: A pot encia 00 n ao possui deni ca o; ela e considerada uma forma indeterminada. Atividade proposta 1.1 Calcule: 4 3 (a) (2)2 (b) 3 (c) 2 Atividade proposta 1.2 Sejam m e n n umeros inteiros e a um n umero real. Mostre que as igualdades abaixo s ao verdadeiras: (a) am+n = am an ; am b) n = amn ; a (c) (a b)n = an bn a n an = n , para b = 0 (d) b b

12.2

Pot encia de expoente racional


recordaremos algumas

Antes de denirmos pot encias de expoentes racionais, propriedades dos radicais.

Deni c ao: Sejam a e b n umeros reais e n um n umero inteiro positivo. Dizemos que n O n umero n a = b, quando bn = a

a e chamado raiz n- esima de a. 3 8 = 2, porque 23 = 8.

Exemplo:

Propriedades operat orias: Sejam a e b n umeros reais n ao-negativos, e n um n umero inteiro positivo. Ent ao valem as seguintes igualdades: 139

(a)

a b = n a n b; n a a n (b) = , para b = 0. n b b (c) n am = np amp , para todo inteiro positivo p.


n

n n n Demonstra c ao: (a) Seja c = n a b. Ent ao cn = ( n a) b c = n a n b . Donde obtemos o resultado desejado.

. Da segue-se que

n n ( n a) a a n . Ent ao c = n = . Da (b) Seja c = segue-se o resultado desejado. n n b b b n (c) Seja c = n am . Ent ao cn = n am = am . Donde obtemos cnp = amp . Ou seja, c = np amp . Podemos agora denir pot encia de um expoente racional. Deni c ao: Sejam m e n inteiros, com n > 0, e a um n umero real n ao-negativo.

Denimos an =
m

am

Propriedades operat orias: Sejam x e y n umeros racionais. Ent ao valem as seguintes igualdades: (a) ax ay = ax+y ; (b)
ax ay

= axy ;

(c) (a b)x = ax bx ; a x ax (d) = x , b = 0. b b Demonstra c ao: Demonstraremos o item (a), os demais cam como atividade proposta. p r Sejam x = e y = , com q = 0 e s = 0 . Ent ao q s ps+qr p p r r qs a q a s = ax ay = q ap s ar = qs aps sq arq = qs aps aqr = aps+qr = a qs = a q + s = ax+y .

140

12.3

Pot encia de expoente irracional

Consideremos o seguinte problema: como denir a pot encia 3 2 ? A solu ca o desse problema, nos dar a inspira ca o para o caso geral de denir a pot encia a , onde a e um n umero real positivo e e um n umero irracional qualquer. Para denir 3 2 , observamos que o n umero irracional

2 = 1.4142 . . . pode ser

aproximado por duas sequ encias de n umeros: a primeira S1 constitu da de n umeros racionais 1 1.4 1.41 1.414 1.4142 . . . que v ao se aproximando do expoente S2 constitu da de n umeros racionais 2 1.5 1.42 1.415 1.4143 . . . que v ao se aproximando do expoente 3
2

2 por valores menores do que

2; e a segunda

2 por valores maiores do que

2. Denimos ent ao

como sendo o n umero para o qual qualquer uma das sequ encias num ericas 31 , 31.4, 31.41 , 31.414 , 31,4142 , . . .

ou 31 , 3154 , 31.42 , 31.415 , 31,4143 , . . . 141

convergem. (De fato, pode ser mostrado que essas sequ encias convergem; isso voc e ver a num curso de c alculo, num t opico sobre sequ encias num ericas). Generalizando, dados um n umero real a > 0 e um n umero irracional , podemos denir a do seguinte modo: consideramos uma sequ encia de n umeros racionais r1 , r2 , r3,... que v ao se aproximando cada vez mais do expoente por valores menores do que e denimos a como sendo o n umero para o qual a sequ encia ar1 , ar2 , ar3 , . . . Converge. Observa c ao: Posteriormente, em um curso mais avan cado, ser a poss vel mostrar, utilizando as propriedades de limite de uma sequ encia, que se x e y s ao n umeros reais, ent ao valem as seguintes igualdades: (a) ax ay = ax+y ; (b)
ax ay

= axy ;

(c) (a b)x = ax bx ; a x ax (d) = x , b = 0. b b

12.4

Fun c oes exponenciais

Passaremos agora para o estudo das fun co es exponenciais. Deni c ao: Dado um n umero real a tal que a > 0 e a = 1 , chamamos fun c ao exponencial de base a a fun ca o f : R R, f (x) = ax . Exemplos: (a) f (x) = 3x ; (b) f (x) =
1 x ; 4

(c) f (x) =

Exemplo: Determine o dom nio da fun ca o f (x) = (x + 1)2x . 142

Solu c ao: A base da fun ca o exponencial deve ser positiva e diferente de 1; o expoente, por sua vez, pode ser qualquer n umero real. Ent ao, conclu mos que x + 1 > 0 e x + 1 = 1 . Da segue-se que x > 1 e x = 0. Levando-se em considera ca o essas duas restri co es, conclu mos que o dom nio da fun ca o dada e o conjunto {x R | x > 1 e x = 0} . Atividade proposta 1.3: Determine o dom nio da fun ca o: (a) f (x) = (x 1)(x9) ; (b) f (x) = (x2 1)

(x9)

Exemplo: Esboce os gr acos das fun co es: (a) y = 2x ; (b) y = Solu c ao: x y = 2x 1 0.5 0 1 1 2 2 3 4 8
1 x 2

(a)

y 3 2 1 2 1 y = 2x x 1 2

(b) x y = (1/2)x 2 4 1 2 0 1 2 3

1 0.5 1/4 1/8

143

y 3 2 1 1
1 x y = (2 )

x 1 2

Propriedades das fun co es exponenciais: Seja a um n umero real com a > 0 e a = 1. (i) Se f (x) = ax , ent ao f (0) = a0 = 1 ; (ii) Se a > 1, ent ao f (x) = ax e uma fun ca o crescente (isto e, se x2 > x1 , ent ao f (x2 ) > f (x1 )). Para ver isso basta observar se f (x2 ) = ax2 e f (x1 ) = ax1 , ent ao
f ( x2 ) f ( x1 )

= ax2 x1 > 1. Esta u ltima desigualdade e v alida porque a > 1 e x2 x1 > 0.

Da segue-se que f (x2 ) > f (x1 ), como quer amos. (iii) Se 0 < a < 1, ent ao f (x) = ax e uma fun ca o decrescente (isto e, se x2 > x1 , ent ao f (x2 ) < f (x1 ) (verique isto). (iv) f (x) = ax e uma fun ca o injetiva (isto e, tem a seguinte propriedade: se x2 = x1 , ent ao f (x2 ) = f (x1 ) ). Para ver isso, seja x2 = x1 . Se x2 > x1 , ent ao pelos itens (ii) e (iii) acima obtemos f (x2 ) > f (x1 ) . Analogamente, se x1 > x2 , conclu mos pelos itens (ii) e (iii) que f (x1 ) > f (x2 ) . Portanto, em ambos os casos, obtemos f (x2 ) = f (x1 ). Observa c ao: Decorre do item (iv) acima que se f (x2 ) = f (x1 ), ent ao x2 = x1 .

12.5

Equa c oes exponenciais

As equa co es exponenciais s ao equa co es nas quais as inc ognitas se encontram nos expoentes. Nesta se ca o ser ao vistos alguns exemplos desse tipo de equa ca o. Exemplo: Resolva a equa ca o 2x+3 = 32 Solu ca o: Fatorando 32, podemos reescrever a equa ca o dada na 2x+3 = 25 forma . Da segue-se que x + 3 = 5. Logo, x = 2 144

Exemplo: Resolva a equa ca o 3x

2 +3x+3

= 27x+4 .
2 +3x+3

Solu c ao: Fatorando 27, podemos reescrever a equa ca o dada na forma 3x (33 )
x+4

. Desse modo, 3x

2 +3x+3

= 33x+12 . Donde obtemos x2 + 3x + 3 = 3x + 12. Ent ao,

x2 = 9. Logo, x = 3. Exemplo: Resolva a equa ca o 2x + 2x+2 = 10. Solu c ao: Note que podemos reescrever o primeiro membro da equa ca o acima da seguinte maneira: 2x + 2x+2 = 2x + 2x 22 = 2x (1 + 4) = 2x 5. Desse modo, a equa ca o dada pode ser reescrita na forma 2x 5 = 10. Logo, 2x = 2, Donde, x = 1. Atividade proposta 1.4: Encontre as solu co es das equa co es abaixo, caso existam: (a) 25x+1 = 5x4 ; (b) 4x+1 = 16x3; (c) 3x1 + 3x+1 + 3x+2 = 111.

12.6

O n umero e

O n umero e e atualmente muito utilizado nas ci encias aplicadas, como Economia e F sica. Sua origem se encontra estreitamente vinculada, na Matem atica Financeira, ao c alculo dos juros compostos (Veja AVILA, para mais informa co es). O n umero e e o n umero denido como o n umero limn 1 +
1 n ; n

ou seja, e = lim 1+ 1 n
n

(L e-se: o n umero e e o limite de 1 +

1 n n

quando n tende ao innito).

Vamos entender melhor essa deni ca o. Ela est a dizendo que o n umero e e o n umero para o qual os valores das pot encias 1 +
1 n n

se aproximam de e quando os n umeros inteiros n

crescem ilimitadamente, tendendo ao innito. Vamos, ent ao, fazer algumas contas para tentar ter uma no ca o do valor desse n umero e: Para n = 1, o valor de 1 +
1 n n

e 1+

1 1 1

= 2;

145

para n = 2, o valor de 1 + para n = 3, o valor de 1 +

1 n n 1 n n

e 1+ e 1+

1 2 2 1 3 3

= =

9 4

= 2.25;
4 3 3

64 27

= 2, 3704 . . . ; = 2, 5937424601 . . . ; 1+
1 100 100

para n = 10, o valor de 1 + para n = 100,

1 n n

e 1+

1 10 10

11 10 10

o valor de

1+

1 100 100

101 100 100

2, 7048138294215260932671080753 . . . . Poder amos continuar esses c alculos atribuindo valores cada vez maiores a n, contudo essa n ao seria uma boa estrat egia para a determina ca o das propriedades do n umero e. Em vez disso, passaremos a fazer algumas observa co es mais produtivas: (1) Para cada valor inteiro que atribuirmos a n, o valor correspondente 1 + um n umero racional; (2) As contas feitas acima n ao garantem que os n umeros 1 + algum n umero ou n ao; (3) As contas feitas acima n ao revelam se o n umero e e racional ou irracional. Contudo, em alguns livros de c alculo diferencial voc e pode encontrar a demonstra ca o de que o n umero e e um n umero irracional cujo valor e, aproximadamente, 2, 718 (Veja, por exemplo, GUIDORIZZI, Hamilton. Um Curso de C alculo). Recomendo que voc e leia tamb em o breve artigo de Elon Lages encontrado no site http://meusite.mackenzie.com.br/giselahgomes/arquivos/numero e.pdf no qual voc e encontrar a mais informa co es acerca do n umero e e sua rela ca o com os logaritmos, assunto da nossa pr oxima aula.
1 n n 1 n n

de ser a

ir ao se aproximar de

12.7

Inequa c oes exponenciais

Exemplo: Determine x de modo que: (a) (x 4) 3x < 0; (b) (x2 1) ex2 > 0. Solu c ao: (a) O problema dado consiste em determinar x de modo que o produto (x 4) 3x seja 146

negativo. Note que nesse produto o fator 3x e sempre positivo. Ent ao o sinal do produto coincidir a com o sinal do fator (x 4). Mas a fun ca o am e negativa para x < 4 e positiva para x > 4 . Portanto, o produto (x 4) 3x ser a negativo, para x < 4; e ser a positivo para x > 4. (b) O problema dado consiste em determinar x de modo que o produto (x2 1) ex2 seja positivo. Note que nesse produto o fator ex2 e sempre positivo. Ent ao o sinal do produto (x2 1) ex2 coincidir a com o sinal do fator (x2 1). Mas a fun ca o x2 1 quadr atica e negativa para 1 < x < 1 e positiva para x < 1 ou x > 1. Logo o produto dado (x2 1) ex2 ser a positivo para e negativo para x < 1 ou x > 1. Atividade proposta: Estude o sinal das seguintes fun co es: (a) f (x) = (x + 5) 4x1 ; (b) f (x) = (x2 2x + 3) e(x1) .

147

Aula 13 Fun c ao Logar tmica


Passemos agora ao estudo da fun ca o logar tmica. Iniciaremos, na se ca o 6.1, recordando o conceito de logaritmo de um n umero e algumas de suas propriedades fundamentais, para em seguida, na se ca o 6.2, passar ao estudo da fun ca o logar tmica.

13.1

O logaritmo de um n umero

Deni ca o: Dados dois n umeros reais a e b, tais que a > 0, a = 1 e b > 0, chama-se logaritmo de b na base a a ` solu ca o x da equa ca o exponencial ax = b . Ou seja, loga b = x se, e somente se, ax = b. Na express ao loga b = x, dizemos que a e a base do logaritmo, b e o logaritmando e x eo logaritmo de b na base a. Exemplos: (a) log3 9 = 2, porque 32 = 9 (b) Como 53 = 125, ent ao log5 125 = 3. Exemplo: Calcule: (a) log2 32 ; (b) log 1 4.
4

Solu c ao: (a) fa ca log2 32 = x . Ent ao 2x = 32 = 25 . Donde segue-se que x = 5. (b) Fa ca log1/4 4 = x. Essa igualdade, pela deni ca o de logaritmo, e equivalente a ` equa ca o exponencial
1 x 4

= 4 . Donde obtemos 2x = 22 . Logo, x = 2. 148

13.2

Algumas conseq u encias importantes da deni c ao de logaritmo

(i) loga 1 = 0 para todo n umero real a > 0 e a = 1; ou seja, o logaritmo do n umero 1 em qualquer base a e igual a 0. Para ver isso, basta fazer loga 1 = x. Da obtemos ax = 1. Logo, x = 0. (ii) loga a = 1, ou seja, o logaritmo da base na pr opria base e igual a 1. (iii) aloga = b, ou seja, a pot encia de base a e expoente loga b e igual a b. Essa arma ca o decorre imediatamente da deni ca o de logaritmo, haja vista que o logaritmo de b na base a e o expoente que se deve dar a ` base a para que a pot encia obtida seja igual a b. (iv) loga b = loga c, ent ao aloga c = b. Para ver isso, suponha que loga b = loga c . Ent ao, pela deni ca o de um logaritmo, conclu mos que aloga c = b . Dessa u ltima igualdade e da propriedade anterior (iii) segue-se que c = b.
b

13.3

Propriedades operat orias do logaritmo


b c

(i) Logaritmo do produto: loga (b c) = loga b + loga c; (ii) Logaritmo do quociente: loga = loga b loga c; (iii) logaritmo de uma pot encia: loga br = r loga b. Demonstra c ao da propriedade (i): Fa ca x = loga b, y = loga c, z = loga b c Ent ao, pela deni ca o de logaritmo ax = b, ay = c, az = b c. Desta u ltima igualdade conclu mos que az = b c = ax ay = ax+y . Logo, z = x + y . Ou seja, loga b c = loga b + loga c, que e o resultado que quer amos. Atividade proposta 1.5: Demonstre as propriedades (ii) e (iii) acima. (Sugest ao: para demonstrar o item (ii) fa ca x = loga b, y = loga c, z = loga para demonstrar o item (iii) fa ca x = loga b, y = loga br ).
b c

149

Atividade proposta: Sabendo que loga b = 3, loga c = 2 calcule: (a) loga (b2 c3 ) ; (b) loga
b 10 c

(c) log a2 b3 c4 .

13.4

Fun c ao logar tmica

Deni c ao: Dado um n umero real a (a > 0, a = 1) a fun ca o logar tmica de base a ea fun ca o

f : (0, +) R, f (x) = loga x, que a cada n umero real positivo x associa o n umero real loga x, o logaritmo de x na base a. Exemplo: Determine o dom nio da fun ca o f (x) = logx (x2 4). Solu c ao: Pela deni ca o de logaritmo, devemos ter a base do logaritmo positiva e diferente de 1; al em disso, o logaritmando deve ser positivo. Desse modo, devemos ter as seguintes desigualdades: x > 1 e x = 0; x2 4 > 0 . Esta u ltima desigualdade e equivalente a x < 2 e x > 2 . Levando em considera ca o todas essas restri co es, conclu mos, ent ao, que o dom nio dessa fun ca o e o conjunto {x R | x > 2}. Atividade proposta 1.6: Determine o dom nio das fun co es: (a) f (x) = log2 (x2 x) ; (b) f (x) = logx1 (x2 x) . Exemplo: Esboce os gr acos das fun co es: (a) y = log2 x ; (b) y = log1/2 x. Solu c ao:

150

(a)

x y = log2 x

1/8 1/4 1/2 1 2 3 y 2 1 0 1

4 8 2 3

y = log2 x x 1 2 3

1 2 (b) x y = log1/2 x 1/8 1/4 1/2 1 3 y 3 2 1 y = log 1 x


2

8 3

1 2

x 1 1 2 2 3

13.5

Propriedades:

(i) Se a > 1, ent ao f (x) = loga x e uma fun ca o crescente. Para ver isso, basta observar que x2 > x1 aloga x2 > aloga x1 , pelo item (iii) da se ca o 6.2 acima. Logo,
aloga x2 aloga x1

> 1; donde segue-se que . Como aloga x2 loga x1 > 1, conclu mos que

loga x2 loga x1 > 0. Logo, loga x2 > loga x1 . O que mostra o que quer amos. (ii) se 0 < a < 1, ent ao loga x e uma fun ca o decrescente. Para ver isso, basta observar que x2 > x1 aloga x2 > aloga x1 ent ao , pelo item (iii) da se ca o 6.2 acima. Logo, 151
aloga x2 aloga x1

> 1;

donde segue-se que . Como 0 < a < 1, conclu mos que loga x2 loga x1 < 0. Logo, loga x2 < loga x1 , o que mostra o que quer amos.

13.6

Gr acos

(i) Se a > 1 o gr aco de y = f (x) = loga x e crescente e corta o eixo dos x no ponto de abscissa x = 1, tendo o aspecto indicado na gura abaixo: y 1 y = log2 x x 1 1 2 (ii) Se 0 < a < 1 o gr aco de y = f (x) = loga x e decrescente e corta o eixo dos x no ponto de abscissa x = 1, tendo o aspecto indicado na gura abaixo: y 3 2 1 y = log 1 x
2

x 1 1 2 2 3

152

13.7

Logaritmos decimal e natural

Dois tipos de logaritmos se destacam no emprego dos logaritmos: o decimal e o natural. O logaritmo decimal e o logaritmo de um n umero calculado na base a = 10, e o logaritmo natural, por sua vez, e o logaritmo de um n umero calculado na base e = 2, 718 . . ., um n umero irracional que se notabilizou por aparecer na resolu ca o de v arios problemas relevantes, tais como o do c alculo de juros compostos, o da determina ca o da taxa de crescimento populacional, o da desintegra ca o radioativa, o do esfriamento de corpos. O leitor interessado em saber mais sobre esse tema pode consultar livros do ensino m edio ou livros de c alculo 1 (Veja Dante, Iezzi, Lima et al., Avila).

153

Bibliograa
[1] DANTE, Matem atica: contexto e aplica co es, vol. 1. Editora Atica. [2] LIMA, Elon et al. A matem atica do Ensino M edio, vol. 1. SBM. [3] LIMA, Elon et al. Exame de textos: an alise de livros de matem atica para o Ensino M edio SBM. [4] IEZZI, Gelson et al. Fundamentos da matem atica, vol. 2 Editora Atual. [5] AVILA, Geraldo. C alculo 1. Livros T ecnicos e Cient cos.

154

S-ar putea să vă placă și